You are on page 1of 38

CLAT – 02

INSTRUCTIONS TO TEST TAKERS

1. Before using the Question Booklet and OMR Answer Sheet. Check them for any defect like misprint, fudging
of printing, missing pages/Questions/ Ovals etc. and ask for issue of relevant duplicates.
2. No duplicate Questions Booklet, OMR Answer Sheet or Extra Blank Sheets of paper shall be provided except
in a situation under Instruction 1 above.
3. There are FIVE Sections (Section I-V) comprising of 150 Objective Questions in the Question Booklet and
each question carries ONE mark. The answers to these objective questions are to be entered on the
OMR Answer Sheet by fully shading the appropriate ovals with HB Pencil Only. If more than one oval
in shaded in answer to a question, that answer shall be deemed to be wrong.
4. There is +1 for every correct answer and -0.25 for every wrong answer.
5. Specific instructions have been given at the beginning of each section and parts of each section. Read them
carefully before answering.
6. Possession of all kinds of electronic gadgets in the exam hall is strictly prohibited. Possession and / or use of
any unfair means shall disqualify the candidate and decision of the Centre Superintendent in this regard shall
be final.
.

OMR sheet has been attached in the end


CLAT 02 (2022 – 23)
Directions for Questions 1 to 5: Read the passage and answer the questions:
As pandemic restrictions have eased, demand has rocketed and supply has struggled to keep up. The war in
Ukraine has accentuated these pressures. Inflation was already rising well above targets and the world had
begun to re-examine its reliance on global supply chains that can easily be interrupted. Now energy and food
security have become a major policy focus. Whether and how these issues are resolved will impact on the
global economy for years to come.
The International Monetary Fund, which downgraded its main world growth forecast from 4.4 percent to 3.6
percent in 2022 and from 3.8 percent to 3.6 percent for next year, stressed that because of the war higher
inflation would persist for longer. For advanced economies, inflation is forecast at 5.7 percent for 2022, falling
to 2.5 percent in 2023. For emerging markets, it is likely to be worse—at 8.7 and 6.5 percent respectively.
The IMF also estimated that, should there be an oil and gas blockage as a result of the EU imposing extra
sanctions by mid-2022, global inflation would be some 1 percent higher and the Eurozone would see a 3
percent hit to GDP, compared to the baseline. The slower world growth that would follow would then
dampen inflation from 2024 onwards, but gas prices in Europe would still be some 20 percent higher than in
the baseline forecast.
How likely is that second scenario? The tone of the EU seems to be toughening up despite differences
between member states, notably resistance to an early embargo by Germany. Even Germany is now moving
towards banning Russian oil by the end of the year, with gas said to follow. Academics and others are doing
helpful work on how to make any embargo clear the legal hurdles and work in practice. But it will all be brutal
in the short term. Even on the baseline scenario, with current policies being sustained, Germany and Italy are
the two G7 countries facing the biggest growth downgrades this year.
On the positive side, the release of US strategic oil reserves and the slowdown in China have brought oil
prices down from their March peaks. There have also been suggestions, pushed mainly by Italian prime
minister Mario Draghi, to combine forces so that EU countries engage in joint gas procurement, enhancing
their bargaining power and thus reducing the price the EU pays for its gas imports. Joint stockpiling of gas
reserves would also improve energy security. Germany is also finally starting the process of developing
Liquefied Natural Gas facilities to be able to receive shipments from other gas-producing regions. At some
point, supply and demand will hopefully start to work to stabilize prices and also improve energy security.
1. Which of these cannot be inferred from the 1st paragraph of the passage?
a) War has caused disruptions in the supply chains of the world.
b) The demand for goods by the public was suppressed due to the pandemic.
c) Demand for essentials rises the most in distressing times.
d) The fire of the war has been fanned by the ongoing pandemic.
2. Which of these, if true, would not weaken the contention of the author mentioned in the 2nd paragraph?
a) The war has given ample opportunities to the USA and Canada for supplying their surplus oil and gas to the
other nations.
b) Russia, after the commencement of the war, has doubled its oil production, distribution, and transport to
the required countries.
c) The demand for essential goods has exponentially increased due to the fear of war and pandemic.
d) The major logistic companies hail from Europe, and Europe has now broken all its ties with Russia.
3. What is the significance of the last paragraph of the passage?
a) To talk about the ways to counter the dependence on imports of oil.
b) To appraise the measures taken to counter the rising inflation.
c) To bring out the conglomeration made in Europe for the import of natural gas.
d) To ascertain the countries taking measures to secure themselves from war.
4. Which of these would aptly justify the title of the passage?
a) Understanding inflation
b) The aftereffect of war on economies
c) Countering rising inflation
d) Economic insecurity
5. Which of these could be the lines to follow the passage?

2
CLAT 02 (2022 – 23)
a) This may be more difficult to achieve with food security. Sharply rising freight costs even before the war
was creating supply and delivery problems—as were delays at ports, where congestion remains an issue.
b) The head of the World Bank, David Malpass, has warned of a humanitarian food catastrophe across the
globe, and that some 60 per cent of the world‘s poorest countries are facing or could soon face serious debt
problems.
c) The emphasis here is ―at all times.‖ The question therefore is how transitory the current crisis is. If supply
cannot easily respond to higher prices, if input costs are such that the increased expense of planting crops
cannot be passed onto the consumer.
d) No wonder, then, that when the Word Economic Forum published its Global Risks Report to accompany its
virtual Davos annual meeting in January, the experts surveyed put ―livelihood crises‖.

Directions for Questions 6 to 10: Read the passage and answer the questions:
Every education system on earth has the same hierarchy of subjects. Every one. Doesn‘t matter where you go.
You‘d think it would be otherwise, but it isn‘t.
At the top are mathematics and languages, then the humanities, and the bottom are the arts. Everywhere on
Earth. And in pretty much every system too, there‘s a hierarchy within the arts. Art and music are normally
given a higher status in schools than drama and dance. There isn‘t an education system on the planet that
teaches dance everyday to children the way we teach them mathematics. Why? Why not? I think this is rather
important. I think math is very important, but so is dance. Children dance all the time if they‘re allowed to, we
all do. We all have bodies, don‘t we? Did I miss a meeting? Truthfully, what happens is, as children grow up,
we start to educate them progressively from the waist up. And then we focus on their heads. And slightly to
one side.
If you were to visit education, as an alien, and say ―What‘s it for, public education?‖ I think you‘d have to
conclude — if you look at the output, who really succeeds by this, who does everything that they should, who
gets all the brownie points, who are the winners — I think you‘d have to conclude the whole purpose of
public education throughout the world is to produce university professors. Isn‘t it? They‘re the people who
come out the top. And I used to be one, so there.
And I like university professors, but you know, we shouldn‘t hold them up as the high-water mark of all human
achievement. They‘re just a form of life, another form of life. But they‘re rather curious, and I say this out of
affection for them. There‘s something curious about professors in my experience — not all of them, but
typically — they live in their heads. They live up there, and slightly to one side. They‘re disembodied, you
know, in a kind of literal way. They look upon their body as a form of transport for their heads, don‘t they? It‘s
a way of getting their head to meetings.
If you want real evidence of out-of-body experiences, by the way, get yourself along to a residential
conference of senior academics, and pop into the discotheque on the final night. And there you will see it —
grown men and women writhing uncontrollably, off the beat, waiting until it ends so they can go home and
write a paper about it.
Now our education system is predicated on the idea of academic ability. And there‘s a reason. The whole
system was invented — around the world, there were no public systems of education, really, before the 19th
century. They all came into being to meet the needs of industrialism.
So the hierarchy is rooted on two ideas. Number one, that the most useful subjects for work are at the top.
So you were probably steered benignly away from things at school when you were a kid, things you liked, on
the grounds that you would never get a job doing that. Is that right? Don‘t do music, you‘re not going to be a
musician; don‘t do art, you won‘t be an artist. Benign advice — now, profoundly mistaken. The whole world is
engulfed in a revolution.
And the second is academic ability, which has really come to dominate our view of intelligence, because the
universities designed the system in their image. If you think of it, the whole system of public education around
the world is a protracted process of university entrance. And the consequence is that many highly talented,
brilliant, creative people think they‘re not, because the thing they were good at, at school wasn‘t valued, or
was actually stigmatized. And I think we can‘t afford to go on that way.

3
CLAT 02 (2022 – 23)
In the next 30 years, according to UNESCO, more people worldwide will be graduating through education
than since the beginning of history. More people, and it‘s the combination of all the things we‘ve talked about
— technology and its transformation effect on work, and demography and the huge explosion in population.
Suddenly, degrees aren‘t worth anything. Isn‘t that true? When I was a student, if you had a degree, you had a
job. If you didn‘t have a job it‘s because you didn‘t want one. And I didn‘t want one, frankly. But now kids with
degrees are often heading home to carry on playing video games, because you need an MA where the
previous job required a BA, and now you need a PhD for the other. It‘s a process of academic inflation. And it
indicates the whole structure of education is shifting beneath our feet. We need to radically rethink our view
of intelligence.
6. Which of these does not identify the attitude of the author in the passage?
a) The author mocks the hierarchy of the education system.
b) The author views biasedness in the education system as hypocrisy.
c) The author longs for a system of learning that lays equal balance on the body.
d) The author condemns the run of society that envisages everyone as a professor.
7. Which of these does the author fail to illuminate in the passage?
a) The difficulties faced by the artists in living a life that meets the economic ends.
b) Gaining knowledge in a manner that may be suitable people who are willing to become professors.
c) The behaviour which causes one to move towards spheres that predominantly require intelligence
smoothly.
d) The slow lapse of the entire education system, that simply degrades the worthiness it initially offered.
8. What could follow next after the passage?
a) A paragraph that discusses education and changes in the performance evaluation system.
b) An example of a candidate who pursued a PhD and got a job that required the skills of a BA.
c) A restructure of the definition of intelligence to understand it in a broader sense.
d) A paragraph on the evolution of the education system and the decadence in its mechanism.
9. Which of these is an idea expressed in the passage?
a) The dominance of creativity has stigmatized the public education system.
b) The University entrance exams have sidelined the creative skills of many.
c) Children should be given the liberty to be able to make a decisive decision regarding the subjects and things
of choice.
d) Dancing is the best way to relax and ease the mind, yet it is not promoted by many of the academicians nor
by the parents.
10. What should be the title of the passage?
a) Schools kill creativity.
b) Balanced education.
c) Hierarchy in the education system.
d) Education or Creativity.

Directions for Questions 11to 15: Read the passage and answer the question based on it.
The Evergrande Group is China‘s second-largest real estate company in terms of total sales and employs over
200,000 employees. Its core business deals with buying large amounts of land, developing them into houses,
restaurants and so on and selling them to interested buyers. The company uses large amounts of debt from
banks and investors as well as short-term loans extended by suppliers and property buyers to fund its
business.
It has total liabilities worth over $300 billion and has to pay around $37 billion in interest and maturing debt
over the next one year. The company‘s bonds have been downgraded by rating agencies such as Fitch and S&P
and have traded well below 50 cents on the Dollar, given the company‘s precarious financial position.
The company has also taken money in advance from over 1.5 million property buyers, promising to deliver
developed properties to them in the future and is yet to pay many suppliers. The company‘s wealth
management team has collected over $6 billion from its own employees promising high returns. It has

4
CLAT 02 (2022 – 23)
defaulted on these products and has offered to give away parking spaces and other real estate in lieu of these
loans, leading to public outcry.
Why is the company in trouble?
The most immediate trigger of the current crisis, analysts believe, is the Chinese government‘s new rules for
property developers. In August 2020, the Chinese government came up with rules (also called the ‗three red
lines‘) stating how much a property developer can borrow given its financial position as measured by three
debt metrics. The new rules practically cut off Evergrande from taking on any more debt on its balance sheet.
This was a big hit to Evergrande‘s business as it engaged in heavy borrowing to run its business. The company
was thus forced to sell its land and other properties at steep discounts to meet its debt obligations. This fire
sale of assets, it is said, eventually led to Evergrande‘s insolvency.
Some see the Chinese government‘s new rules as a bid to puncture the country‘s property bubble and bring
about a ‗soft landing‘ of the economy. Chinese authorities have traditionally encouraged businesses to take on
huge amounts of debt through the heavily state-controlled financial sector to develop new properties. This led
to the indiscriminate development of properties, so much so that almost a third of the Chinese GDP is made
up of the property sector. Millions of properties with very little demand from buyers have been seen in
Chinese ‗ghost cities‘.
Over the years, Chinese authorities have also tacitly supported the property bubble by bailing out troubled
developers. A letter leaked last year by Bloomberg showed Evergrande‘s executives seeking financial
assistance from the Chinese government by admitting their inability to meet their debt obligations to banks. It
is said that the Chinese authorities now want the country‘s resources to be allocated towards other sectors
such as technology and have thus decided to withdraw their support for property developers. It should be
noted that the Chinese government has in recent years let various business entities with exposure to the
property sector go bust instead of bailing them out of trouble. The restrictions on property developers are
seen by some also as an example of the Chinese government taking on influential private businessmen under
the guise of the ‗common prosperity‘ programme to fight income inequality.
Other analysts, however, believe that the current crisis was a long time coming. They argue that the
company‘s business model has been unsustainable for a long time. In 2012, Andrew Left of Citron Research
argued that Evergrande was insolvent and that the company was engaged in aggressive accounting practices to
cover up its troubles. It was said that the company held properties that it could not sell on its balance sheet as
inventory, thus avoiding the booking of losses. The company was also accused of running a ponzi scheme as it
relied on the constant inflow of funds to prop up a business model that is fundamentally unsustainable.
Interestingly, Evergrande has over the years consistently reported strong profits and has showcased a strong
balance sheet with sufficient liquid assets.
11. Which of these properly illustrates the structure of the 6th paragraph.
a) The government of a country proposes to purchase fighter jets from a local manufacturer than buying them
from another country.
b) The reserve bank lets traders trade in cryptocurrency by tacitly permitting the funds transfer, and then after
a while, it bans trade in cryptocurrency.
c) The principal of a school makes it compulsory for students driving to school to submit a copy of their
licences and allows his children to drive without one.
d) A company allows some of its employees to leave daily at 6 pm while some to leave daily at 7 pm.
12. Which of the following strengthens the contention made by the author in the last paragraph?
a) Roughly 57% of the business of the Evergrande group comes from government projects.
b) In 2018 the company promoters of the company decreased more than 50% their share in the company.
c) The government had previously warned all the real estate companies to set their financials straight.
d) The profits earned by the company were being earned from business activities besides property
development.
13. As per the text of the passage, which of the following is incorrect?
1. Development of property irrespective of the buyers creates a faux of development in the nation.
2. The Chinese government takes decisions based on the financial status of the companies.
3. The fall of Evergrande would affect the lives of more than 0.2 million people in China.
4. The technology sector in China is about to see a boost with governmental support.
5
CLAT 02 (2022 – 23)
a) Only 1
b) 1, 2 and 4
c) 1, 3 and 4
d) All
14. What is the writing style of the author used in the passage?
a) Descriptive
b) Analytical
c) Persuasive
d) Narrative
15. What can be inferred from the passage?
1. The priority of the Chinese government has now shifted from property to technology.
2. The support of the Chinese government to the infrastructure development companies was based on a give
and take relation.
3. The Chinese government now is looking for a bigger opportunity in technology and other sectors that may
yield better financial results.
4. The profits showed by the company in the balance sheet were bogus, and the company was producing fake
accounts and balances in its accounting details.
a) 1 and 2
b) 4 and 2
c) 1 and 3
d) 1 and 4

Direction for Questions 16 to 20: Read the passage and answer the question based on it.
To walk from south to north on the peripatos, the path encircling the Acropolis of Athens – as I did one golden
morning in December last year – takes you past the boisterous crowds swarming the stone seats of the
Theatre of Dionysus. The path then threads just below the partially restored colonnades of the monumental
Propylaea, which was thronged that morning with visitors pausing to chat and take photographs before they
clambered past that monumental gateway up to the Parthenon. Proceed further along the curved trail and, like
an epiphany, you will find yourself in the wilder north-facing precincts of that ancient outcrop. In the section
known as the Long Rocks there are a series of alcoves of varying sizes, named ingloriously by the
archaeologists as caves A, B, C and D. In its unanticipated tranquillity, this stretch of rock still seems to host
the older gods.
I sat below these caves that morning appreciating a respite from the tumult and, for a few minutes, I just
listened. The pursuit of quietness, especially in urban areas has become a preoccupation of mine in recent
years. However, the quiet I experienced below the caves of Zeus Astrapaios, of Apollo and of Pan was not
precisely an encounter with silence, for it was punctuated by many sounds. A family of cats mewled; the wind
gusted playfully across the limestone and the schist, and sent the leaves scuttering along the pavement. A
murmur of voices rose up occasionally from the cafes of the Plaka neighbourhood; someone, somewhere,
played a melancholy air on the klarino. All of these sounds were pleasant to my ears. This form of quietness,
one that is not precisely silence, is characterised rather by an absence of noise or βοή (voe) in Greek, a word
that might also translate as clamour, or din. I call the sort of auditory lull that, at the same time, asserts a
benevolent presence, ‗avoesis‘ (that is, the absence of voe or noise).
After a short time, I moved farther east along the peripatos and the susurrus of idle chatter picked up once
more; a car horn sounded in the distance, and then I discerned the pronounced hum of traffic. A sharp whistle
blew from the top of the Acropolis – I assume a visitor had breached a cordon and had placed a profane foot
upon a protected antiquity. I had now left the quiet behind; my time with the gods was over.
This was my first visit back to Athens in a few decades. The city has always been appealing to me with its
bustling market places; its vendors outside the garrulous cafes cajoling passersby to stop in; the gloaming
sanctuary of its low-domed churches, the hardware merchants outside their stalls immersed in voluble
dialogue (will there be fisticuffs or embraces? … One can never tell, for the arguments never end); the curious
specificity of its engrossing museums; the indefatigability of its derelict buses honking their way through the
snarling streets; the ubiquity of its adventitious feral cats; the sense that poetry has always been possible here;
6
CLAT 02 (2022 – 23)
the lute players and the buskers on the street corners and in the squares; its burdensome heat in summer; its
catastrophic and attention-demanding pavements; its promiscuous mix of wealthy and impoverished streets;
the affability of its winter temperatures; its graffiti: political, amusing and occasionally inane or obscene; the
chestnut vendors on the sidewalks absorbed with their roasting pans; the scattering of its monumental debris;
the alternating mood of despair and vivacity suggestive that both revolution and equanimity are ever-present
possibilities; the dark unkempt verdure of its botanic garden; the reverence Athens has for its past; the sense
that the past should not determine its future.
And then there is the noise, the glorious polyglottic, polyphonic commotion of Athens arising from its people,
vehicles and its infernal construction machinery. I had wondered, at first, if enduring the tumult of the city was
a young person‘s game, and perhaps it is, but returning to Athens seemed like an assignation with an old lover,
whose whispers remain electrifying and whose harsh words are astringent but still exciting. Even so, I longed
for some relief from the city and its din. I left, after a few days, for the mountainous Peloponnese.
16. If given a chance, what would the author like to change about the scene being described?
a) The names given to the caves by the archaeologists.
b) The sound that the author hears in the cave of Zeus Astrapaios.
c) The visitors who visit the Acropolis of Athens.
d) The sellers in the market of Athens.
17. Which of these would the author find analogous with the path to the caves.
a) A journey that goes through the mountains and ends at a waterfall.
b) A walk in the woods that eventually leads to national highway.
c) A drive through a valley that leads towards a city.
d) A busy day of work that leads to a calm weekend.
18. Which of these can be inferred from the passage?
a) Avoesis can be a path to tranquillity with the nature but not Bon voe.
b) The streets of the Athens have not been developed and are still the way how they used to be in the past.
c) Most the visitors who visit Acropolis of Athens have lack decorum as they enter the areas that are restricted
for the visitors.
d) Athens‘ central market place is infamous for its sellers who call cajole people to make purchases.
19. Which of these could be a suitable title to the passage?
a) Silence: The way to tranquillity
b) Athens: The land of Gods
c) A visit to Athens
d) A moment of silence in chaos
20. The author is most likely to be a/an
a) Archaeologist
b) History professor
c) Traveller
d) Literary writer

Direction for Questions 21 to 25: Read the passage and answer the question based on it.
In the historiography of modern India, the renaissance is generally marked as the pre-political phase of the
anti-colonial struggle, a period when Indians were mainly engaged in social and cultural preparation for
participation in the more ―progressive‖ and ―radical‖, political programme. The social and religious
movements, popularly termed as the renaissance, which preceded the political struggles, are considered a
necessary precursor to the coming of nationalism.
This teleological view of history has been dominant till recently. A departure from this view, quite critical for
renaissance studies, had to wait until a strict periodisation of historical time came to be questioned. Not only
broad overarching labels like ancient, medieval, modern and contemporary periods, but also thematic
periodisation like the colonial, reformist or nationalist periods came under scrutiny. The challenge to this neat
compartmentalisation came from different sources. To begin with, from Marxist scholars who traced the
social origins of the national movement, from the marginalised sect of scholars who came out with alternative

7
CLAT 02 (2022 – 23)
histories based on caste, and subaltern historians whose focus was on domination and subordination. This not
only marked a change in the universe of analysis, but also a reconceptualisation of categories and the re-
examination of analytical categories such as caste, class, community, and so on. In the realm of the history of
ideas, the intellectual history, if you like, the most important departure has been the contextualisation of ideas.
The relationship between modernity and the renaissance has given rise to a variety of questions. Whether the
renaissance succeeded in resolving the social contradictions that existed in society is one important question.
Why the renaissance did not become trans-sectional and why it remained religion-caste oriented is another. Is
it that the renaissance was the expression of nothing more than an aggregation of elite-caste social and
religious interests? Is it a fair assessment that the renaissance did not succeed in transgressing the limits set by
the ideologies of just one favoured caste? Is it accidental that the university syllabuses did not contain courses
on the history of the marginalised? Why did the historical literature on the evolution of modern India treat the
renaissance as an overarching phenomenon striding across the Indian society in the 19th and 20th centuries,
without much sensitivity to the fortunes of the marginalised? An inquiry into the relationship between
renaissance and modernity may provide answers to some of these questions.
The origin of modernity in India is often attributed to the intellectual and cultural efflorescence associated with
the renaissance. The renaissance marked a period of transition in values, transformation in social sensibilities
and rebirth in cultural creativity. The outcome of these processes was the elaboration, representation and
interpretation of humanism and the emergence of a new man with cultural and intellectual attributes different
from his past. These ideas inspired an upsurge of creative energy, leading to the works of masters in painting,
sculpture, literature, music, and so on. The new aesthetic that emerged was integral to the structural
transformation of social organisation and relations of production. It was the intellectual component of the rise
of capitalism, which came to be christened as modern, to distinguish the present from the past—the new from
the old.
21. Which of the following can be inferred as per the passage?
1. Nationalism is conceptualised as a natural outcome of the renaissance.
2. Renaissance and nationalism are independent concepts.
3. The nationalism that led to the Indian independence was directly at loggerheads with the renaissance.
4. The social movement in the country was an essential forerunner in the development of the concept of
nationalism.
a) 1 and 2
b) 1, 2 and 3
c) 2 and 4
d) 1 and 4
22. If the departure in the contextualisation of ideas is considered to be true, which of the following can be
attributed to it as a reason?
1. The study of the non-social origins of the national movement.
2. The analysis of modern concepts and contemporary periods as well as of the reformists and nationalist
periods.
3. The ‗progressive‘ and ‗radical‘ political programmes that later shaped nationalism.
4. The different histories given by scholars of a particular sect.
a) 1,2 and 3
b) 2 and 4
c) 1,3 and 4
d) 1 and 3
23. Which of the following is a possible inference made in the passage?
a) The elite had their hands tarnished by reaping the best for themselves, which was later observed by
intellectuals.
b) The history of the evolution of modern India imperially vitiated the elite sections of the society.
c) The concept of modernity and renaissance was formed in the latter half of the 19th century.
d) The period prior to the renaissance was a time that changed the shape of nationalism and increased its
ambit.
24. Which of the following, if true, would strengthen the contention of the author made in the passage?
8
CLAT 02 (2022 – 23)
a) There was a rise in the number of schools that made a paradigm shift from pseudo-centric learning to
creative learning at the time of the renaissance.
b) There was an increase in the number of scholars due to a change in the oppression that was marked in the
prehistoric era.
c) The extensive growth in the theory of capitalism was inversely related to the fall in the renaissance, which
was later cited as modernism.
d) The mark of the new man, from the viewpoint of the scholars, was an innuendo to the rise of the era of
capitalism in the period of renaissance.
25. The structure of the last two paragraphs can be analogous to which of the following?
a) The emergence of the Y2K virus with the birth of the 21st century that eventually led to the birth of the
future of technology.
b) The conundrum of the replacement of electric vehicles with the rise in oil prices but with the expense of
disposing more battery waste into the world.
c) The infusion of a vaccine that initially was considered problematic but later, many progressive signs showed
up.
d) A scheme of the government that was made to provide food to children in schools but was later adjusted as
per the availability of resources and logistics.

Directions for Questions 26 to 30: Read the passage and answer the question based on it.
Regulatory networks are increasingly important actors in global and regional arenas. Organised in communities
centred on shared technical interests and common areas of practice, networks can play a role at all stages of
policy innovation, especially within specialised sectors such as finance, environment, or human rights. They can
offer innovative solutions and causal explanations for policy choices and support governments to identify their
preferences and interests. Networks can be a valuable source of information and mediation between
international and regional regulators, such as the United Nations (UN) and the European Union (EU) and
national governments as targets of international policy and often are successful in influencing the form and
content of national and regional policy outcomes.
The realm of human rights promotion and protection is no exception. In recent decades, the human rights
regime has transformed to accommodate a dense network of state and non-state actors that operate through
increasingly complex relationships at the bilateral, multilateral, regional, and trans-governmental level. Existing
scholarship has provided ample evidence about the important role that regulatory networks have played in the
diffusion of human rights norms across borders, shaping these norms and supporting the efforts to monitor
their implementation. Transnational networks, such as the Global Alliance of National Human Rights
Institutions (GANHRI) and its regional counterparts offer good examples, as they are network-based
regulatory structures made up of national regulatory bodies mandated to promote and protect human rights –
national human rights institutions (NHRIs) – that have been key to the continued development of their
institutional members. NHRIs and their peer networks function as intermediaries in a global order in which,
generally, international and regional (in Europe) organisations are the main international regulators of
international human rights law and national governments are primary targets showing different degrees of
compliance international law.
In recent decades, NHRIs have become increasingly important institutional actors in Europe. Described as
‗domestic non-judicial institution[s] for the implementation of human rights law‘, NHRIs are regulatory
intermediaries that serve to embed international norms in local structures. As the primary independent
domestic bodies charged with the implementation of international human rights treaties ratified by
governments, human rights ombudsmen and human rights commissions play the dual role of promoting and
protecting rights at the national level. NHRIs are key to countries‘ efforts to address adequately a range of
rights-related issues like the protection and promotion of fundamental rights, social and political rights.
In recent years, NHRIs in Europe have organised to formalise the development of a consolidated regional
network of peer bodies, the European Network of National Human Rights Institutions (ENNHRI). Founded in
1994, the European Group of NHRIs met irregularly until 2002 under the auspices of the Council of Europe
(CoE) and relied on its administrative and financial support to continue its informal activity. The network
consolidated its powers and formalised its centralised administration in an international context in which
9
CLAT 02 (2022 – 23)
resistance to human rights has been on the rise and funding cuts to human rights institutions are commonplace.
What explains the recent rapid development of ENNHRI as a regional regulatory network of human rights
institutions in a global context that is increasingly less favourable to human rights?
26. Which of the following would be a case aptly suited for NHRIs to intervene?
a) An attack by a terrorist organisation on a school full of children, leading to a public outcry.
b) A cut done by the government in the rescue package offered to the families hit by the flood to divert money
for the purchase of defence warfare.
c) The rejection of a mercy plea by a court of law on the grounds of setting an exemplary model.
d) The quelling of unrest caused by a mob - motivated by a faction - having views contrary to the state welfare.
27. With context to the main idea of the passage, what is the role of the 1st paragraph?
a) To illustrate the role of relations and networks and to bring to light the necessity of fundamental rights.
b) To understand the importance of networks that assist NHRIs and the quiddity of reasonable assistance
offered by them.
c) The need for NHRIs in the sub context of Europe for electoral stability with the support of the local,
national and international connections.
d) The consistent network building on the sidelines of the violation of fundamental rights and the work of
NHRIs.
28. Which of these is a possible inference made by the author in the passage?
1. The formation of a centralised body shall have more power and influence to press fundamental rights.
2. Not only bodies with the power to direct rights but also with no legislative or authoritative powers are
necessary for the protection of socio-political rights.
3. Nexus with the layers of the governing bodies and local connections embarks the dispersion of human rights
a cut above discrete endeavour.
4. Innate solutions to a place can come forth with bona fide links that are aboriginal to a place.
a) Only 3
b) 1, 2 and 3
c) 2, 3 and 4
d) 1 and 2
29. Considering a tree that absorbs water from the roots and provides it to its leaves, which of the following is
analogous to various parts of a tree, keeping in mind the flow of water, when compared to the context of the
passage?
a) (i) Roots - NHRIs (ii) Trunk – Public (iii) Leaves – Network
b) (i) Roots - Network (ii) Trunk - NHRIs (iii) Leaves - Public
c) (i) Roots - Public (ii) Trunk – NHRIs (iii) Leaves – Network
d) (i) Roots - NHRIs (ii) Trunk – Network (iii) Leaves – Public
30. What is the attitude of the author towards network in the passage?
a) The author lauds the regulatory networks and voices their importance.
b) The author pin-points their use of networks and explains their work.
c) The author exaggerates the importance of regulatory networks and offers his opinion on them.
d) The author values regulatory networks and shows his displeasure in their working.

Direction for Questions 31 to 35: Read the passage and answer the following questions.
The first batch of five Rafale jets flew out of France on July 25 and arrived in India on Wednesday, July 29,
when the fighter jets will officially be inducted and join the Indian Air Force fleet in Haryana's Ambala. The
fighter jets, built by French aviation firm [1], took off from the Merignac airbase in southern France's Bordeaux
today and landed safely at Al Dhafra airbase in the UAE after a sortie of more than seven hours. The five
aircraft will be the first tranche of the 36 planes bought by India from France in a Rs. 59,000-crore inter-
governmental deal in 2016. ―Delivery of ten aircraft has been completed on schedule. Five will stay back in
France for training Mission. The delivery of all thirty six aircraft will be completed on schedule by the end of
2021, the Indian embassy in France said in a statement. The Rafale fighter jets are capable of carrying a range
of highly effective weapons, including the Meteor air-to-air missile and Scalp cruise missile. The Rafale jets will

10
CLAT 02 (2022 – 23)
come with various India-specific modifications, including Israeli helmet-mounted displays, radar warning
receivers, low-band jammers, 10-hour flight data recording, infra-red search and tracking systems among
others. The Air Force has readied the required infrastructure to welcome the jets in its line-up.
Source: Excerpt from the NDTV, Reported by Vishnu Som, Edited by Shylaja Varma.
Link: (https://www.ndtv.com/india-news/rafale-fighter-jets-first-batch-to-fly-out-of-france-today-2269361)
31. Which of the following has been replaced by [1] in the passage?
a) LAMSCO
b) LUAD Defence France
c) DASSAULT
d) BORESCOPE
32. Which of the following organisations/institutions is a major offset partner of India in the Rafale Deal?
a) Hindustan Aeronautics Limited
b) Reliance Defence Limited
c) Reliance Naval and Engineering Limited
d) Pipavav Defence
33. The Indian Air Force is also in the process of procuring a new generation medium-range modular air-to-
ground weapon system Hammer to integrate with the Rafale jets. What does the term Hammer stands for?
a) Highly Advanced Modular Mutation Extended Range.
b) Highly Agile Modular Munition Extended Range.
c) Highly Advanced Modular Mitigation Extended Rafale.
d) Highly Agile Modular Munition Extended Rafale.
34. Which of the following indigenous organisations is responsible for the design, assembly and fabrication of
combat aircrafts in India?
a) DRDO
b) HAL
c) DRAI
d) ISRO
35. The Rafale deal between India and France is seen to carry great strategic significance. Which of the following is
directly impacted by this deal?
a) The Joint Strategic Vision of India-France Cooperation in the Indian Ocean Region.
b) The one belt one road mission of the China.
c) The Chinese initiative to build a new Silk route.
d) The trade route of the Middle East.

Direction for Questions 36 to 40: Read the passage and answer the following questions.
The Centre on Thursday significantly reduced the footprint of the Armed Forces Special Powers Act (AFSPA),
1958 in the Northeast, withdrawing it entirely from [1] districts in Assam; and partially from seven districts in
Nagaland, six districts in Manipur, and one district in Assam. Once the decision is notified in the gazette,
AFSPA remains in force in parts of these three states as well as in parts of Arunachal Pradesh and Jammu &
Kashmir. AFSPA, which has been called draconian, gives sweeping powers to the armed forces. For example,
it allows them to open fire, even causing death, against any person in contravention to the law or carrying
arms and ammunition, and gives them powers to arrest individuals without warrants, on the basis of
―reasonable suspicion‖, and also search premises without warrants. It can be imposed by the Centre or the
Governor of a state, on the state or parts of it, after these areas are declared ―disturbed‘‘ under [2]. The
Northeast has lived under the shadow of AFSPA for nearly 60 years, creating a feeling of alienation from the
rest of the country. The move is expected to help demilitarise the region; it will lift restrictions of movements
through check points and frisking of residents.
https://indianexpress.com/article/explained/afspa-law-and-the-northeast-nagaland-manipur-assam-7846909/
36. Which of the following will replace [1] in the above passage?
a) 18
b) 23

11
CLAT 02 (2022 – 23)
c) 28
d) 20
37. Which of the following will replace [2] in the above passage?
a) Section 5
b) Section 2
c) Section 4
d) Section 3
38. Recently, the armed forces killed 14 villagers of which of the district of Nagaland?
a) Tuensang Wokha
b) Phek
c) Mon
d) Dimapur
39. Which of the following is TRUE about AFSPA?
a) The Justice Jeevan Reddy Commission submitted its report in 2008, saying AFSPA had become a symbol of
oppression and recommending its repeal.
b) The UPA government rejected the findings of the Reddy Commission.
c) The Fourth Administrative Reforms Commission, headed by Veeerapa Moily, endorsed these
recommendations.
d) Manipur activist Irom Sharmila began a hunger-strike, which would continue for 16 years, against AFSPA.
40. Which of the following is TRUE?
a) Tripura revoked the Act in 2015.
b) AFSPA was revoked in Meghalaya in 2018
c) AFSPA was withdrawn from Punjab in 1997
d) All of these

Direction for Questions 41 to 45: Read the passage and answer the following questions.
On May 8, India‗s Defence Minister virtually inaugurated a new 80 km-long road in the Himalayas, connecting
to the border with China, at the Lipulekh pass. The Nepali government protested immediately, contending
that the road crosses territory that it claims and accusing India of changing the status quo without diplomatic
consultations. Among the many escalatory moves since then, Nepal deployed police forces to the region,
summoned the Indian ambassador in Kathmandu, and initiated a constitutional amendment to formalise and
extend its territorial claims over approximately 400 sq km. India, on the other hand, has conveyed its
openness to a dialogue but does not seem to share Nepal‗s sense of urgency: its initial statement agreed to a
dialogue, but only after the COVID-19 crisis. India has been in effective possession of this territory for at least
sixty years, although Nepal claims it conducted a census there in the early 1950s and refers to the 1815 Sugauli
Treaty as legitimising its claims. But India‗s new road, up to the Lipulekh pass, is not an unprecedented change
in the status quo. India has controlled this territory and built other infrastructure here before, besides
conducting its administration and deploying military forces up to the border pass with China. The region is of
strategic importance, and the new road is now one of the quickest links between Delhi and the Tibetan
plateau. In a 2015 statement, China also recognised India‗s sovereignty by agreeing to expand trade through
the Lipulekh pass.
Source: Excerpt from the brookings.edu, written by Dr. Constantino Xavier.
Link: (https://www.brookings.edu/blog/up-front/2020/06/11/interpreting-the-india-nepal-border-dispute/)
41. Recently Nepal has issued a map claiming Lipulekh pass as part of its territory included in the district of:
a) Kailali
b) Darchula
c) Dipayal
d) Doti
42. The confusion between Nepal and India consists of a dispute over a trijunture of ____________.
a) Kali – Susta – Ganga.
b) Kalapani – Saraswati – Ganga.

12
CLAT 02 (2022 – 23)
c) Kalapani – Limpiyadhura – Lipulekh.
d) Kalapani – Limpiyadhura – Kali.
43. The treaty of Sugauli was signed between which of the following pairs?
a) India and Nepal
b) United Kingdom, India and Nepal
c) India, China and Nepal.
d) East India Company and Nepal
44. Which of the following rivers is called as Narayani River in Nepal?
a) Bramhaputra
b) Gandak
c) Ganga
d) Kali
45. Which of the following incidents can be seen as the direct cause of the border dispute between India and
Nepal?
a) The Defence Minister inaugurated a motor-able link road that connects India and China.
b) The 2015 agreement between India and China for using the Lipulekh pass for trade.
c) India published a new map which showed the region of Kalapani as part of the Indian Territory.
d) All of these

Direction for Questions 46 to 50: Read the passage and answer the following questions.
Rajasthan chief minister Ashok Gehlot on Monday said Prime Minister Narendra Modi should announce the
revival of the old pension scheme (OPS) for employees across the country as done by the Rajasthan
government. ―Modi and other states will have to restore the OPS, they have no other choice,‖ said Gehlot
adding that after the announcement for restoring the old pension scheme in the Rajasthan state budget, the
Chhattisgarh government has also declared the same.
The chief minister‘s statement came before the Rajasthan assembly passed the state budget by voice vote on
Monday. Replying to the appropriation bill, he also made many announcements including discontinuation of [1]
monthly deduction from the salary of state government employees for the new pension scheme from April 1,
2022.
https://www.hindustantimes.com/cities/jaipur-news/restore-old-pension-scheme-for-employees-ashok-gehlot-
to-pm-modi-101647898037022.html
46. Which of the following will replace [1] in the above passage?
a) 15%
b) 12%
c) 20%
d) 10%
47. The New Pension Scheme is NOT applicable on which of the following?
a) Teachers
b) Armed Forces
c) Politicians
d) Judges
48. Which of the following is NOT TRUE about the New Pension Scheme?
a) The Central Government introduced the National Pension System (NPS) with effect from January 2004.
b) The All Citizens Model of the NPS allows all citizens of India excluding NRIs aged between 18 - 70 years to
join NPS.
c) On introduction of NPS, the Central Civil Services (Pension) Rules, 1972 was amended.
d) NPS is being implemented and regulated by Pension Fund Regulatory and Development Authority
49. Which of the following is TRUE about the Old Pension Scheme?
a) The scheme assures life-long income, post-retirement.
b) Usually the assured amount is equivalent to 50% of the last drawn salary.
c) The Government bears the expenditure incurred on the pension.

13
CLAT 02 (2022 – 23)
d) All of these
50. In which of the following years did the Finance Ministry said that Central government employees have the
option of selecting the Pension Funds (PFs) and Investment Pattern under the New Pension Scheme?
a) 2012
b) 2019
c) 2014
d) 2017

Direction for Questions 51 to 55: Read the passage and answer the following questions.
One thing struck us as a major difference between the new National Education Policy (NEP) 2020 and its
predecessor. The earlier national policies on education (NPE) from 1986 and 1992 presented themselves as
attempts to consolidate and build on earlier efforts, particularly the NPE, 1968. The new NEP 2020 policy, on
the other hand, is very keen to establish that it is different from everything in the past, including in its name.
Nowhere does this attitude come across as starkly as it does in the section on higher education. It comes
across fairly clearly on how the higher education ecosystem will be by 2040. By this time — if the policy has its
way — the Indian higher education ecosystem will be populated with higher education institutions (HEI).
These will comprise Universities and Colleges and the public and private sectors, all of which will be multi-
disciplinary, with each populated by more than 3,000 students, with at least one ―in or near every district.
Universities will conduct research and post-graduate and under-graduate teaching, some research-intensive
and others teaching-intensive. Colleges will largely teach at the under-graduate level, with a number of them
having their medium of instruction in either bilingual or local Indian languages. The colleges can manifest in
clusters around universities as constituent colleges or may be standalone autonomous ones. Ideally, all HEIs
will eventually become ―independent self-governing institutions with considerable ―faculty and institutional
autonomy. They will have complied with a series of regulatory exercises that are ―light-but-tight and will be
operated by a large number of private accreditors, overseen by a new set of regulatory institutions at the
national level.
Source: Excerpt from downtoearth.org.in, written by Nitin Mehta & Gagan Mehta.
51. What is the proposed percentage of expenditure for education out of GDP under the NEP 2020?
a) 11%
b) 6%
c) 4.6%
d) 9%
52. Which of the following is proposed as a change in the structure of institutions responsible for affiliation and
accreditation under the NEP 2020?
a) Higher Education Commission of India (HECI) will be set up as a single umbrella body for the entire higher
education.
b) University Grant Commission to be renamed as AICTE.
c) National Affiliation and Accreditation Council to be renamed as National Standard Council of India.
d) National Council for Education Research and Teaching to be renamed as BCERT.
53. Which of the following is not a vertical under the Higher Education Commission of India (HECI)?
a) National Higher Education Regulatory Council (NHERC)
b) Higher Education Grants Council (HEGC)
c) National Educational Council (NEC)
d) National Accreditation Council (NAC)
54. Which of the following statements are not true in relation to the changes envisioned by the NEP 2020?
a) The NEP 2020 aims at making India a global knowledge superpower.
b) The renaming of the Ministry of Human Resource Development to the Ministry of Education.
c) The development of National Course Curriculum to provide new curriculum by 2021.
d) The New Education Policy aims to facilitate an inclusive, participatory and holistic approach.
55. Which of the following is proposed as a change in the structure of institutions responsible for affiliation and
accreditation under the NEP 2020?

14
CLAT 02 (2022 – 23)
a) Higher Education Commission of India (HECI) will be set up as a single umbrella body for the entire higher
education.
b) University Grant Commission to be renamed as AICTE.
c) National Affiliation and Accreditation Council to be renamed as National Standard Council of India.
d) National Council for Education Research and Teaching to be renamed as BCERT.

Direction for Questions 56 to 60: Read the passage and answer the following questions.
Nearly 40 years after India first signed the Antarctic Treaty, the government has brought in a draft Indian
Antarctic Bill-2022 to regulate and monitor activities at its research stations in the frozen continent.
Introducing the draft Bill in the Lok Sabha on Friday, Earth Sciences Minister [1] said the primary aim of the Bill
is to bring in penal provisions for breaking the law in Antarctica. It has a comprehensive list of permitted
activities on the continent and prohibitions, he said.
The Opposition members opposed the Bill. ―The Bill is applicable to Indian citizens as well as foreign citizens.
How can an Indian law be applicable on foreign citizens? How can you punish a foreign citizen if he or she
commits an offence under the law? This is my first question,‖ said Congress MP Adhir Ranjan Chowdhury,
demanding that the Bill be referred to the Select Committee or Joint Committee.
https://indianexpress.com/article/india/40-yrs-after-treaty-antarctic-bill-introduced-in-ls-7848788/
56. Which of the following will replace [1] in the above passage?
a) Harshvardhan
b) Ravi Shankar Prasad
c) Jitendra Singh
d) Kiren Rijuju
57. Which of the following is NOT TRUE about Antarctic Treaty?
a) India became a member of this treaty in 1999.
b) Antarctic Treaty was signed in 1959.
c) It entered into force in 1961.
d) None of these
58. Which of the following is the first Indian scientific research base station established in Antarctica?
a) Maitri
b) Bharti
c) Dakshin Gangotri
d) Sagar Nidhi
59. Which of the following is TRUE about Antarctica?
a) Antarctica is Earth's southernmost continent.
b) It contains the geographic South Pole and is situated in the Antarctic region of the Southern Hemisphere.
c) At 14,000,000 square kilometers, it is the fifth-largest continent.
d) All of these
60. How many countries are signatory to the Antarctic Treaty?
a) 54
b) 26
c) 46
d) 62

Direction for Questions 61 to 65: Read the passage and answer the following questions.
The longstanding dispute between Punjab and Haryana over Chandigarh flared up this week after the Centre
notified Central Service Rules for employees in the Union Territory instead of the Punjab Service Rules. The
Centre had earlier tweaked rules for appointments to the Bhakra Beas Management Board (BBMB) —
recruitments can now be done from anywhere in India, instead of just Punjab and Haryana. On Friday, the
Punjab Assembly in a special session passed a unanimous resolution reiterating the state‘s claim on Chandigarh.
After Partition, Shimla was made the temporary capital of Indian Punjab. Prime Minister Jawaharlal Nehru
wanted a modern city to replace Lahore as Punjab‘s capital, and the idea of Chandigarh was conceived. In

15
CLAT 02 (2022 – 23)
March [1], the Punjab government, in consultation with the Centre, chose the picturesque foothills of the
Shivaliks as the site of the new capital. Twenty-two villages in Kharar were acquired for the city, and the
government compensated their displaced residents. The capital was officially moved from Shimla to
Chandigarh on September 21, [2]. President Rajendra Prasad inaugurated the new capital on October 7, [2].
Until Haryana was born, Chandigarh remained the capital of Punjab.
https://indianexpress.com/article/explained/explained-punjab-resolution-chandigarh-capital-7848602/
61. Which of the following will replace [1] in the above passage?
a) 1947
b) 1949
c) 1950
d) 1948
62. Which of the following will replace [2] in the above passage?
a) 1951
b) 1955
c) 1956
d) 1953
63. Which of the following is NOT CORRECT?
a) According to the Capital of Punjab (Development and Regulation) Act, 1952, the properties in Chandigarh
were to be divided in a 70:30 ratio in favour of Punjab.
b) In 1970, the Centre announced that ―the capital project area of Chandigarh should, as a whole, go to
Punjab‖.
c) Haryana was told to use the office and residential accommodation in Chandigarh for five years until it built
its own capital.
d) None of these
64. The Rajiv-Longowal Accord was signed between then Prime Minister Rajiv Gandhi and Akali leader Harchand
Singh Longowal in which of the following years?
a) 1984
b) 1988
c) 1985
d) 1991
65. The State Reorganisation Act of Punjab carved Haryana out of Punjab in which of the following years?
a) 1966
b) 1960
c) 1963
d) 1970

Direction for Questions 66 to 70: Read the passage and answer the following questions.
Several courts have used privacy as a justification to restrict speech without employing any balancing test. The
Delhi High Court asked a publisher to remove infringing content from a book about the petitioner, as it
violated the petitioner‘s fundamental right to privacy. Rather ironically, the Court provided extracts of the
same text in its judgement. In another judgement given by the Madras High Court, the Court not only
employed the fundamental right horizontally, but it also went a step further by stating that the right to
privacy would have a privileged position over other rights after Puttaswamy.
The Supreme Court in affirming the fundamental right of privacy provided a safeguard from government
violation of individual privacy, however, it did not answer questions with regards to the horizontal
applicability of the right. Across the world, there has been a call to bring even private players under the ambit
of the fundamental right of privacy, given the gargantuan amounts of data that many technology companies
hold. But holding the right to be horizontally applicable could also have far-reaching consequences on the
rights of a free press.

16
CLAT 02 (2022 – 23)
As has happened in the cases of Swami Ramdev and Kanimozhi, enforcing the fundamental right of privacy
against private players could mean subjecting media houses to expensive litigation, making them even more
vulnerable when several of them already run on shoestring budgets.
This is not to say that news outlets are free to publish private information about anyone, even public
personalities, carelessly. There have been cases where news outlets have stepped beyond their boundaries
bringing blameless individuals or their private lives into the public spotlight by publishing their personal details.
However, these cases cannot be used as a means to undermine the importance of journalism in bringing
attention to matters of public import. There have been several instances where publishing private information
about a few individuals was necessary. This ranges from releasing sensitive financial information about people
who use law firms in Panama to private telephonic conversations between lobbyists and Union ministers.
Extracted from Rakhecha V, ‘Privacy, the Shield That Protects or the Sword That Strikes Freedom
of Speech?’ (2021) 5 Indian Law Review 62
66. The author discusses the horizontal applicability of the right to privacy in the passage as highlighted above in
bold. Which of the following are examples of horizontal application of rights?
a) Non-procedural deprivation of liberty is prohibited by the Indian Constitution. A, a Member of the Jammu
and Kashmir Legislative Assembly, was arrested by the police without following the law‘s procedure due to
political motivations. A filed a suit against the police claiming compensation.
b) Untouchability is prohibited by the Indian Constitution. A was practising untouchability against B. B filed
initiated legal proceedings claiming that his constitutional right has been violated by A.
c) Sex-based discrimination is prohibited by the Indian Constitution. The government made laws prohibiting all
females from working in places that serve liquor. A, a female, claimed her rights in Court by filing a suit against
the government.
d) All of the Above.
67. The author here wishes to strike a balance between persons‘ rights to privacy and the reporting leeway which
can be justifiably given to the press. Would this line of reasoning be in consonance with a situation wherein a
journalist enters the house of an indigent person without their consent, clicks photographs of their house all
around and publishes them in the newspaper in order to show the state of poverty in India?
a) Yes, an intrusive privacy violation is acceptable in circumstances where it subserves the wider public good.
b) No, since it was not necessary to enter one‘s house non-consensually to document their state of living and
publish it, making the privacy violation unjustifiable.
c) Yes, since it would be in the interests of the public to acquire information about the implementation of
government schemes and their failure.
d) No, since the indigent person do not come under the category of ―lobbyists and Union Ministers‖ as
discussed in the last paragraph.
68. A is a civil servant who has been alleged to be in a nexus with a big businessman X, who wishes for him (i.e.,
A) to approve documents that would accord him (i.e., X) monopoly rights over the supply of electricity in his
city. B, a journalist, acquires knowledge of the same and publishes the accusations in the following day‘s
newspaper. Since these were mere accusations and B did not have legitimate evidence of the same, X sued B
and his company for Rs. 100 crores, which the Court awarded, pedestalizing X‘s right to privacy. Would the
Court‘s award be deemed acceptable by the author?
a) No, since the author believes in utilitarian thought and would want the greatest good of the greatest
number to be consistently achieved. The nexus between the civil servant and the businessman is an obstacle to
the same, and thus the author would support a blanket waiver of their privacy interests.
b) Yes, since the author believes in according reverence to litigants‘ privacy rights, regardless of adverse
consequences that might follow.
c) No, since the author believes that there exist certain circumstances where the publication of private
information is necessary in order to subserve the public good. Since this is a matter that concerns the public at
large, privacy interests of the two can be undermined.
d) Yes, since the author believes in the utilitarian thought and the presence of natural monopolies such as
those on electricity would ultimately drive costs down and prices down, which would lead to the public good
being served.

17
CLAT 02 (2022 – 23)
69. The Madras High Court has stated in a judgment that privacy rights shall have a privileged position after
Puttaswamy in the first paragraph. If the author were to accord precedential value to this Madras HC
judgement, would your answer to the above question change?
a) Yes, since the privacy rights of X and the A shall now be protected at all costs, given that they now enjoy a
privileged position.
b) No, since Madras HC‘s interpretation of Puttaswamy is resulting in immediate injustice, since the
businessman and civil servant can take actions against public interest.
c) Yes, since Puttaswamy‘s fundamental purpose was to protect big businessmen and civil servants from
unnecessary litigation. The Madras HC reaffirms this and therefore the answer to the above question would
change.
d) No, since the present status of the necessity justification must prevail over the Madras HC‘s judgement.
70. In the present case, a media house showed on TV that a retired judge of the Supreme Court was implicated in
a murder case, without doing any due diligence or research activities on their part. This was broadcast the
entire day, with people glued to their TV screens. Later, it turned out that someone else who shared the same
name as the judge was implicated in the murder case. Thus, the retired judge, feeling extremely embarrassed,
took his matter to Court and sued the media house for Re. 1 in order to get judicial validation of his
innocence. Would the author deem this acceptable?
a) Yes, since the retired judge had legitimate rights to sue the media house but chose a path of benevolence by
suing them only symbolically for Re. 1.
b) No, since the retired judge should understand the thin budgets on which media houses run and abandon the
idea of suing them.
c) Yes, since the author is in favour of disproportionate emphasis on privacy rights and won‘t wish to abandon
them in any circumstance.
d) None of the Above.

Direction for Questions 71 to 75: Read the passage and answer the following questions.
In contrast with general territorial law (which applies equally to everyone), ‗personal law‘ is the body of law
that applies different rules to different religious or ethnic communities. During colonial rule, general territorial
laws were enacted (and interpreted by the courts) as the British Raj consolidated its power. However, the Raj
chose to preserve religion-specific personal law rather than replace it with uniform law that applied across
communities.
About eighteen months after the Constitution came into force, the Bombay High Court was confronted with
an important case involving personal law. Several Hindu men were charged with offences of bigamy under
Bombay‘s bigamy law – the Bombay Prevention of Hindu Bigamous Marriages Act of 1946. This law not only
made bigamous marriages invalid among Hindus, but also made it a criminal offence (punishable with up to
seven years in prison) for those that entered into such marriages.
The law was challenged on two grounds. First, it was argued that the law violated the right to profess, practise
and propagate religion under Article 25 of the Constitution. The court found it relatively easy to reject this
argument. Article 25 also permitted the state to provide for ‗social welfare and reform‘. As Chief Justice
Chagla noted, large sections of society considered monogamy ‗a very desirable and praiseworthy institution‘ –
a move towards monogamy would therefore constitute social reform.
It was argued that once the Constitution came into force, Muslim personal law permitting polygamy as well as
Hindu personal law permitting bigamy was inconsistent with fundamental rights under the Constitution and,
therefore, invalid. The inconsistency arose from the fact that it discriminated between men and women, by
permitting Muslim and Hindu men (but not women) to have more than one spouse. Therefore, even though
the Constitution changed the status quo for both Hindu men and Muslim men, the Bombay law only applied to
Hindus. In these circumstances, the Bombay law was unconstitutional for singling out and criminalizing bigamy
among Hindus and excluding Muslims.
This argument was reliant on the language of Article 13 of the Constitution, which was part of the chapter on
fundamental rights. Under Article 13, ‗[a]ll laws in force in the territory of India‘ immediately before the
commencement of the Constitution would be void to the extent that they were inconsistent with fundamental
rights. Did uncodified personal law count as ‗law in force‘ under Article 13? This question assumed particular
18
CLAT 02 (2022 – 23)
significance extending beyond the Narasu case. If the court held that personal law was not ‗law in force‘, then
all uncodified personal law would effectively be immune from legal challenge on the grounds that it violated
fundamental rights.
Extracted from State of Bombay v. Narasu Appa Mali in ―The Cases that India Forgot‖ by Chintan
Chandrachud
71. Which of the following, pursuant to the characteristics laid down in the first paragraph, constitute personal
law?
a) Rules dealing with personalized ideas such as those of agency, autonomy, dignity and destiny of persons.
b) Rules that differentiate their application based on persons‘ ages: some rules for minors, other ones for
majors.
c) Rules that draw distinctions based on persons‘ religious identities, some laws apply to people from A
religion, others apply to people from B religion.
d) All of the Above
72. What would be the consequences on the law‘s validity if the two arguments presented in the fourth paragraph
were accepted by the Court, the first on differentiating on sex and the other on religion?
a) It would be unconstitutional on grounds of its limited application to Hindus and leaving Muslims out,
regardless of the sex-based discriminatory aspect.
b) It would still be constitutionally valid, personal laws already have an immune status and any attempt to
interfere would be disastrous.
c) It would be declared invalid on both grounds on challenge: sex-based discrimination and religion-based
classification since both are grounds of challenge presented in the argument.
d) It will stay constitutionally compatible since a comparison can be drawn between uncodified Muslim law and
the Bombay Bigamy Act. If the first can stay compatible and yet permit polygamy, there is no basis to declare
the latter bad in law.
73. You are given that the right to free speech and expression is a fundamental right guaranteed in the
Constitution. There was a colonial-era law that criminalized all publications that are deemed so (i.e. criminal)
by an area‘s District Magistrate. A district‘s DM created a local gazette, an extract from which reads the
following: ―Publications dealing with political matters, Indian government and politics, questions of civil rights,
foreign relations of India, matters of public policy, matters of sports shall not be published on any social
platform, newspaper, magazine, SMS, WhatsApp and similar entities….‖ The law on the basis of which this
rule was framed was challenged by concerned citizens. Would it be compatible under Article 13, as discussed
in the fifth paragraph?
a) Yes, since it would be incorrect to deem the law itself invalid on the basis of an unfair use by an area‘s DM.
Merely because its application is unfair, the law itself does not become invalid.
b) No, since it is clear that the effect of the given law is such that it vitiates the fundamental right to free
speech and expression. Since it is violating a constitutional principle, it shall not be compatible to the extent
that does.
c) Yes, since the purpose of Article 13 is to protect colonial-era laws to ensure a degree of continuity between
the operation of law before and after Indian independence. Thus, even if it might be violative of constitutional
principles, Article 13 cannot come to its rescue.
d) None of the Above.
74. If the Bombay High Court ruled in Narasu that no personal law shall qualify as a ―law in force‖ for Article 13
purposes, which of the following would be constitutionally invalid? You are given that the Constitution
prohibits discrimination on the grounds of sex, religion, class, caste and place of birth.
a) A personal law of religion A that forbids women from entering a site of religious worship because of their
gender identity.
b) A personal law of religion B that deprives women from seeking maintenance after divorce because of their
gender identity.
c) A personal law of religion C that permits untouchability and promotes discrimination against persons from
marginalized communities on the basis of their caste identities.
d) None of the Above.

19
CLAT 02 (2022 – 23)
75. If the Bombay High Court would have ruled that personal laws constitute ―laws in force‖ for Article 13
purposes, which of the following would be constitutionally incompatible? You are given that the Constitution
prohibits heterosexuality, monogamy and marriage in its fundamental rights section, as discussed in the last
paragraph of the passage.
a) A personal law of religion A that mandates that all members of its community must never have a same-sex
relationship, meaning that heterosexuality is enforced by the religion‘s personal law.
b) A personal law of religion B mandates that all its members shall only have one partner to engage with at a
point of time, meaning that monogamy is mandated.
c) A personal law of religion C states that its members shall be married, meaning that the marital institution is
compelled by the religion‘s personal law.
d) All of the Above

Direction for Questions 76 to 79: Read the passage and answer the following questions.
While the right to health has not been textually guaranteed as a fundamental right in the Indian Constitution, it
has by now been firmly entrenched as such. Through a series of progressive pronouncements delivered in the
last two decades, starting from the 1995 case of Consumer Education and Research Centre v. Union of India,
the Supreme Court has recognised the right as flowing from Article 21 of the Constitution.
In Navtej Johar, when a 5–judge bench of the Supreme Court decriminalised homosexual intercourse, in his
concurring opinion, Justice Chandrachud pertinently held:
―Article 21 does not impose upon the State only negative obligations not to act in such a way as to interfere
with the right to health. This Court also has the power to impose positive obligations upon the State to take
measures to provide adequate resources or access to treatment facilities to secure effective enjoyment of the
right to health.‖
Further, the court has held that financial difficulties cannot come in the way of making medical facilities
available to the people. On March 30, 2020, the Supreme Court urged the government to consider a
representation to make more testing labs and quarantine centres available to deal with COVID–19 cases.
It is also worth noting that an upper limit of Rs 4,500 has been imposed on conducting COVID 19 tests. As the
Delhi high court has held, ―Just because someone is poor, the State cannot allow him to die. After all, health is
not a luxury and should not be the sole possession of a privileged few.‖ As a result, the government will have
to closely monitor the extent to which this price is reasonable, and if evidence emerges of it being an
impediment in accessing tests, courts should step in to help set the situation right.
Source: https://thewire.in/law/can-the-judiciary-invoke-right-to-health-to-demand-a-more-vigorous-response-
to-covid-19
76. Which of the following statements correctly describes the status of the right to health?
I. The right to health is a creation of the judiciary, which was never intended by the Constitution makers.
II. The right to health has always been understood to be inherent in the right to life under Article 21.
III. Right to health can only be considered a fundamental right when the Parliament makes a requisite
amendment in the Constitution.
a) Only I
b) Both II and III
c) Both I and II
d) None of the above
77. Which of the following was not a ruling of the Supreme Court in Navtej Johar?
I. Fundamental rights, in general, not only impose negative obligations but also positive obligations upon the
State.
II. In order that people may enjoy the right to health, it is imperative that the State carry out some steps to
ensure access to resources and facilities.
III. The judiciary has no authority to direct the State to carry out any measures or steps.
a) Only III
b) Both I and II
c) Only I
d) Both I and III
20
CLAT 02 (2022 – 23)
78. The direction of the Supreme Court urging the government to ―consider a representation to make more
testing labs and quarantine centres available to deal with COVID–19 cases‖ can be best described as:
a) An order of the Supreme Court directing the government to carry out its duties.
b) A reprimand of the Supreme Court directed at the government for failing to meet its constitutional
obligations.
c) An instance of the Supreme Court imposing positive obligations on the government.
d) An amendment of the Constitution to add a new Fundamental Right.
79. Arun migrated to Delhi for partaking in the construction industry. But all construction work was stopped due
to the pandemic. A government scheme was announced whereby all the people who had been rendered
jobless were to be helped. Arun decided to walk home and collapsed as soon as he tried. Which of the
following statements can be said to be true?
I. The government carried out its positive obligations under Article 21.
II. The government was not obligated to help Arun.
III. Arun‘s right to health was violated.
IV. The government should have ensured that the transport facilities had enough health equipment.
a) Both I and IV
b) Both II and III
c) Both I and II
d) None of the above

Direction for Questions 80 to 84: Read the passage and answer the following questions.
Frustration is an English contract law doctrine that acts as a device to set aside contracts where an
unforeseen event either renders contractual obligations impossible, or radically changes the party's principal
purpose for entering into the contract. Historically, there had been no way of setting aside an impossible
contract after formation; it was not until 1863, and the case of Taylor v Caldwell, that the beginnings of the
doctrine of frustration were established. Whilst the doctrine has seen expansion from its inception, it is still
narrow in application; Lord Roskill stated that it is "not lightly to be invoked to relieve contracting parties of
the normal consequences of imprudent bargains".
Destruction of the subject matter
Where an item or building essential to the contract – which has been expressly identified – is destroyed,
through no fault of either party, it can be set aside as impossible to perform, as established in Taylor v
Caldwell. Such principles differ however when considering the sale of goods. The agreement of the parties is
important when considering whether it has been frustrated. If it is agreed that goods from a specific source
will be provided, the contract falls under Section 7 of the Sale of Goods Act 1979:
―Where there is an agreement to sell specific goods and subsequently the goods, without any fault on the part
of the seller or buyer, perish before the risk passes to the buyer, the agreement is avoided.‖
However, where a contract does not provide 'specific' goods, as required for the Act to operate, it will fall
under common law rules. A contract will not be frustrated if generic goods are destroyed or rendered
commercially non-viable. The risk is assumed to pass with the seller.
Supervening illegality
Where a law subsequent to contracting is passed, which renders the fundamental principle of contracting
illegal, the contract will be found to be frustrated. There are several situations in which this may occur. Events
such as war may render certain trading or actions illegal, as was the case in Denny, Mott & Dickinson v James
Fraser and Ertel Bieber and Co v Rio Tinto Co Ltd [1918] AC 260. If the law changes prohibit performance
after the contract was made, the contract may be frustrated. Changes in the law may render building work
illegal, or the use of certain materials illegal. A contract for the construction of a reservoir was held to be
frustrated following wartime building regulations.
Such principles apply equally where contractual obligations become illegal in foreign countries, if this is where
they are to occur.
[Source: https://en.wikipedia.org/wiki/Frustration_in_English_law#Frustrating_events]
80. X was a meat-lover. However, being convinced by some of the arguments raised by vegans, he decided he
wanted only free-range chicken as they are kept in a more humane, kind condition than chicken usually
21
CLAT 02 (2022 – 23)
available in the market. Consequently, he approached Y, the owner of the only free-range chicken farm in the
country, and entered into an agreement pursuant to which Y was required to provide 1kg of chicken meat to
X every month. In lieu of the same, X agreed to pay Rs. 10,000 every month, a sum that was rather high
keeping in mind that the price of non-free-range chicken is around Rs. 500 per kg. Owing to a disease in the
chickens at Y‘s farm caused by the poultry-equivalent of food poisoning caused by poor quality of chicken-
feed, Y was not able to provide chicken to X. Can Y avoid paying damages?
a) Yes, because the contract stood frustrated when the chicken contracted the poultry-equivalent of food
poisoning
b) No, because Y was negligent
c) Yes, because Y was giving up on such a high payment – there surely must have been an important reason
behind it
d) No, because there was no supervening illegality
81. In the facts of the question above, assume that the farm burnt down due to a lightning strike as a result of
which Y was not able to provide chicken to X. Would X have been able to successfully sue Y?
a) No, because the contract stood frustrated
b) Yes, because the contract stood frustrated
c) Yes, because Y could have procured similar chicken from elsewhere to supply to X
d) No, because Y was negligent
82. In the facts of the first question, assume that X was not able to pay the pre-decided sum of money to Y
because of his expenditure that month on some accident and health related exigencies, such as spending a lot
of money on having his leg and hip fixed after a serious motorcycle accident. Would Y have been able to
successfully sue X?
a) No, because the contract stood frustrated
b) Yes, because the contract was not frustrated
c) Yes, because X could have, among other things, borrowed money to pay Y
d) Both (b) and (c)
83. A Russian supplier of wheat, X, had a contract with the Indian government pursuant to which he was expected
to deliver a pre-decided amount of wheat to a certain port in India every year. However, owing to sanctions
imposed by India on all Russian traders following the Russian-Ukraine war, he was not able to deliver the
same. Can the Indian government claim damages?
a) No, because the Indian government made the contract impossible to perform itself and not due to a
supervening act/ event
b) No, because the contract stood frustrated because of impossibility of performance
c) No, because the contract was vague to begin with owing to uncertainty in time and place of delivery
d) No, because the contract was vague to begin with owing to uncertainty in price to be paid for the wheat
84. Which of the following kinds of clauses can help mitigate the risk of frustration of contract?
a) Entire Agreement Clause
b) Severability Clause
c) Force Majeure Clause
d) Indemnification Clause

Direction for Questions 85 to 88: Read the passage and answer the following questions.
In jurisprudence, distinction between theoretical and practical aspects of law is the basis of an independent
science of law, the purpose of which is not to subserve practical ends but to serve pure knowledge, which is
concerned with facts but not with words. At the present time, the juristic science is an exclusively a practical
science of law and adequate methods have been evolved for the application of law by the judges. The result of
this situation is that its teaching on the subject of law and legal relations, subject matter and method, can be
given only by the practical science of law. It aims to supply the judge with legal propositions, formulated in the
most general terms possible, in order that the greatest possible number of decisions might be derived from
them. It teaches the judge how to apply the general propositions to the specific cases. However, the human
thinking is necessarily dominated by the underlying purpose and the thinking of the jurist is conditioned by the
practical purposes pursued by juristic science.
22
CLAT 02 (2022 – 23)
The jurist does not mean by law that which lives and is operative in human society as law, but law exclusively
important in the administration of justice as a rule according to which the judges must decide the legal
disputes. However, juristic science as a whole proceeds by abstractions and deductions but sometimes loses
contact with reality. The rule of human conduct and the rule according to which the judges decide legal
disputes may be distinct; a layman does not always act according to the rules which the judges apply for the
judicial decisions as the rules to guide human conduct. However, the scientific view has given way to the
practical view, adapted to the requirements of the judicial officials according to which they must proceed, but
they arrive at this view by a jump in their personal thinking. They mean that the rules according to which
courts decide are the rules according to which men ought to regulate their conduct. In this respect it is
altogether different from true science. It is true that that judicial decisions influence the conduct of men, but
we must first of all inquire to what extent this is true and upon what circumstances it depends.
Source: Fundamentals of Jurisprudence - The Indian Approach, Prof. S.N. Dhyani
85. The purpose of the ‗science of law‘ is to achieve
a) Practical aspects for human conduct.
b) Idealistic judicial abstractions.
c) Pure knowledge based on reality.
d) Judicial purpose.
86. The prevailing method of the practical science of law is the method which is ______
a) Employed by judges for application of law.
b) Indicated in law books.
c) Abstraction of social mind.
d) Legal optimism.
87. The judicial thinking is prominently guided by
a) Underlying principles of judicial morality.
b) Principles of professional ethics.
c) Practical purposes followed by judicial science.
d) Classical theories of justice.
88. The judicial expectation from a layman is that
a) The conduct of man ought to be humane.
b) The conduct of man ought to be prudent.
c) The conduct of man be based on rules decided by courts.
d) The conduct of man be based on moral and ethical parameters.

Direction for Questions 89 to 93: Read the passage and answer the following questions.
Section 299 of the Indian Penal Code defines Culpable Homicide as ―Whosoever causes death by doing an act
with the intention of causing death or with the intention of causing such bodily injury as it is likely to cause
death or with the knowledge that he is likely by such act to cause death, commits the offence of Culpable
Homicide.‖
The essentials of Culpable Homicide are:-
Whoever causes death – Death means death of a human being. It does not include the death of an unborn
child, but it may amount to culpable homicide to cause death of a living child if any part of the child has been
brought forth. However it‘s not necessary that the person whose death has been caused must be the very
person whose death was intended.
(a) Intention to cause death – Intention means the expectation of the consequence in question. When a
man is charged with doing the act of which the probable consequence maybe highly injurious, the intention is
inferred from the acts of the accused and circumstance of the case.
(b) With the intention of causing such bodily injury as is likely to cause death – The intention of the
offender may not be to cause death, it would be sufficient if he intended to cause such bodily injury which was
likely to cause death.

23
CLAT 02 (2022 – 23)
(c) With the knowledge that he is likely by such an act to cause death – Knowledge is a strong word
and imports a certainty and not merely a probability. Here knowledge refers to the personal knowledge of the
person who does the act.
Section 300 of the Code defines murder as – Except in cases such as those with grave and sudden
provocation, culpable homicide is murder if –
1. If the act by which the death is caused is done with the intention of causing death.
2. If it is done with the intention of causing such bodily injury as the offender knows likely to cause death of the
person to whom the harm is caused
3. If it is done with the intention of causing such bodily injury to any person and the bodily injury intended to
be inflicted is sufficient in the ordinary course of nature to cause death
4. If the person committing the act knows that it is so immediately dangerous that it must in all probability
cause death or such bodily injury as is likely to cause death and commits such an act without an excuse for
incurring the risk of causing death or such bodily harm
As in case of Culpable Homicide, intention and knowledge for committing the act is important. Where a man
intentionally inflicts bodily injury sufficient in the ordinary cause of nature to cause death, then such a person
will be liable for murder in case of murder the probability of death is more than in the case of culpable
homicide.
89. Behrun and his father were on bad terms with the appellants. Behrun was going towards his fields when he
was caught hold of by Anda. They dragged him inside a house and beat him severely. His arms and legs were
smashed and many bruises and lacerated wounds were caused on his person. The doctor who conducted the
autopsy opined that injuries and that all the injuries collectively were sufficient in the ordinary course of nature
to cause death. Are the appellants guilty of murder or culpable homicide?
a) They are not guilty because the doctor who conducted autopsy said that the deceased died of shock.
b) They are guilty of culpable homicide, because there is no indication of use of lethal weapon.
c) They are not guilty of murder because intention is missing.
d) It‘s a clear cut case of murder
90. On one day, X was followed by some people. He was not allowed to walk away. The accused pounded and
beat him by heavy sticks and continued the beating till he became unconscious. The accused than left the spot.
The doctor found as many as 19 injuries out of which no less than 9 were (internally) found to be grievous.
The doctor gave the opinion that the injuries were cumulatively sufficient to cause death in the ordinary
course of nature.
a) The offence is murder
b) It is culpable homicide because the intention of the accused is missing.
c) It is neither murder nor culpable homicide.
d) It is a case of simple fight
91. In a case there was a long standing dispute between the deceased and the appellants over a certain piece of
land. On one night, the appellants (N and R) in pursuance of a conspiracy to commit murder of M, deceased,
set fire to the single room hut in which he was sleeping after locking the door from outside. When the
servants of the deceased and other villagers tried to rescue the deceased they were kept at bay by the
superior force of the accused and their associates. Consequently, death of the deceased occurred.
a) It is not a murder, because the facts and circumstances are not sufficient.
b) It is not culpable homicide too because it is nowhere evident that the very action of the accused to set the
house of the deceased on fi re, resulted in to his death.
c) It is murder
d) It will depend on the discretion of the court.
92. Bachhan Singh, son of the deceased, had caused a severe injury on the leg of one Pritam Singh resulting in the
amputation of leg of one appellant. Father of Pritam Singh harboured a grudge against Bachhan Singh and his
father. On one day, the appellant and his associate caught hold of the deceased (father of Bachhan Singh) and
inflicted as many as 18 injuries on the arms and legs of the deceased with a gandasa.
a) It is culpable homicide not amounting to murder.
b) It is murder.
c) It is neither murder nor culpable homicide.
24
CLAT 02 (2022 – 23)
d) Cannot be determined
93. Virsa Singh thrust a spear in the abdomen of the deceased with such force that it penetrated the bowels and
three coils of the intestines came out of the wound. The doctor who conducted the post mortem said that the
injury was sufficient to cause death in the ordinary course of nature.
a) It is a murder
b) It is culpable homicide.
c) It is grievous hurt.
d) None of the above

Direction for Questions 94 to 98: Read the passage and answer the following questions.
Vicarious liability is a form of a strict, secondary liability that arises under the common law doctrine of
agency, respondeat superior, the responsibility of the superior for the acts of their subordinate or, in a
broader sense, the responsibility of any third party that had the "right, ability or duty to control" the activities
of a violator. It can be distinguished from contributory liability, another form of secondary liability, which is
rooted in the tort theory of enterprise liability because, unlike contributory infringement, knowledge is not an
element of vicarious liability. The law has developed the view that some relationships by their nature require
the person who engages others to accept responsibility for the wrongdoing of those others. The most
important such relationship for practical purposes is that of employer and employee.
Employers are vicariously liable, under the respondeat superior doctrine, for negligent acts or omissions by
their employees in the course of employment (sometimes referred to as 'scope and course of employment').
To determine whether the employer is liable, the difference between an independent contractor and an
employee is to be drawn. In order to be vicariously liable, there must be a requisite relationship between the
defendant and the tortfeasor, which could be examined by three tests: Control test, Organisation test, and
Sufficient relationship test. An employer may be held liable under principles of vicarious liability if an employee
does an authorized act in an unauthorized way.
Employers may also be liable under the common law principle represented in the Latin phrase, qui facit per
alium facit per se (one who acts through another acts in one's own interests). That is a parallel concept to
vicarious liability and strict liability, in which one person is held liable in criminal law or tort for the acts or
omissions of another
Courts sometimes distinguish between an employee's "detour" vs. "a frolic of their own". For instance, an
employer will be held liable if it is shown that the employee had gone on a mere detour in carrying out their
duties, such as stopping to buy a beverage or use an automated teller machine while running a work-related
errand, whereas an employee acting in their own right rather than on the employer's business is undertaking a
"frolic" and will not subject the employer to liability.
[Source: https://en.wikipedia.org/wiki/Vicarious_liability]
94. While conducting a search and seizure operation, income tax officer X negligently dropped a cigarette that he
did not properly put out. As a result, the property of the person whose house was being raided, Y, was
severely damaged. Can Y sue the income tax department?
a) Yes, as X is an employee of the income tax department who committed an unauthorised act
b) Yes, as X is an employee of the income tax department who committed an authorised act in a wrongful
manner
c) Yes, as X is an employee of the income tax department who merely took a detour
d) Yes, as X is an employee of the income tax department who was told by the income tax department to set
fire to Y‘s property
95. In the facts of the question above, would the conclusion change if X had been instructed by the income tax
department to set fire to Y‘s house?
a) No, because respondeat superior would apply
b) No, because the concept of ―frolic of their own‖ would not apply
c) No, because qui facit per alium facit per se would apply
d) No, because the concept of contributory infringement would apply
96. The facts of the first question represent which of the following concepts?

25
CLAT 02 (2022 – 23)
i. Respondeat superior
ii. Qui facit per alium facit per se
iii. Contributory liability
iv. Contributory infringement
a) i, ii, iii
b) i, ii
c) i, ii, iv
d) i
97. X had been instructed to pick up his employer from his (the employer‘s) house at 8:00AM. While X was
driving towards the employer‘s home in the morning, X stopped the car to purchase a bottle of liquor for his
own consumption. Due to the negligent manner in which the car had been parked, an accident took place,
injuring Y. Can Y successfully sue X‘s employer?
a) No, because X is not an employee at all
b) Yes, because X merely took a detour
c) No, because the purchase of the liquor bottle for X‘s own use and hence amounted to a ―frolic‖ of his own
d) Yes, because X is unlikely to have the money to compensate Y
98. Which of the following is/ are correct?
a) Secondary liability is a superset and vicarious liability is its subset
b) Secondary liability is a subset and vicarious liability is its superset
c) Secondary liability and vicarious liability are equivalent to each other
d) Secondary liability and vicarious liability have only a few things in common and neither is a subset of the
other

Direction for Questions 99 to 105: Read the passage and answer the following questions.
The ―Right to Information‖ is commonly known as ―RTI.‖ The right to information is a part of the fundamental
right of freedom of speech and expression, enshrined under Article 19(1) of the Constitution of India. The
Right to Information has been made a statutory right through the Parliament‘s enactment, the Right to
Information Act, 2005. This Act is often termed as revolutionary, as a common man can demand any
government agency to furnish information by using this right. Further, this Act mandates timely response to
citizens‘ requests for government information.
Section 2(j) of the RTI Act defines ―right to information,‖ which means the right to information accessible
under this Act which is held by or under the control of any public authority.
Rights available under RTI Act:
 Seek any information which is held by any public authority.
 Take copies of government documents.
 Inspect works, documents, records of government.
 Take notes, extracts, or certified copies of government documents or records.
 Take certified samples of Government work.
 Obtain information in the form of diskettes, floppies, tapes, video cassettes, or in any other electronic
mode or through printouts where such information is stored in a computer or in any other device.
As per Section 2(f) of the RTI Act, 2005, ‗information‘ means material in any form, including records,
documents, memos, e-mails, opinions, advice, press releases, circulars, orders, logbooks, contracts, reports,
papers, samples, models, data material held in any electronic form and information relating to any private body
which a public authority can access under any other law for the time being in force.
According to this definition provided by the RTI Act, citizens can seek information from any Public Authority.
The power given to the citizens under the RTI Act is limitless. Yet, the Public Authority can refuse details
sought through an RTI application if the refusal is not against public interest and the information sought pertain
to national security and defense or some personal information.
It must be noted that opinions cannot be sought under the RTI Act.

26
CLAT 02 (2022 – 23)
As mandated by the RTI Act, all Public Authorities are required to appoint their Public Information Officer
(PIO), who shall be bestowed with the responsibility of dealing with the public‘s requests and providing them
the necessary information as specified under the RTI Act.
Following points to be considered while seeking information under RTI.
 A request seeking information regarding any public authority, may be submitted by any person in writing
to the Public Information Officer.
 It is the duty of the PIO to provide the information sought within 30 days of the receipt of the request.
 When the request is made pertaining to any another public office whether in whole or part, it is the
responsibility of the PIO to transfer the concerned request to a PIO of appropriate authority within 5
working days.
 Also, there is a proviso to Section 7(1) of the RTI Act which states if information has been sought for
concerning the life or liberty of a person, the same shall be provided within 48 hours of the receipt of the
request.
In the Central Act, Section 8(1) has provided an inclusive list of all the exemptions, which can be utilised by a
PIO to reject an RTI application provided that he shall not be entitled to do so in the event that furnishing
requested information subserves the public interest. Following is a general discussion of the exemption
provisions:
 National Security/Sovereignty
 National Economic Interests
 Relations with the Foreign States
 Law Enforcement and the Judicial Process
 Cabinet and Other Decision-Making Documents
 Trade Secrets and Commercial Confidentiality
 Individual Safety
 Personal Privacy
Source: https://blog.ipleaders.in/laymans-guide-to-the-right-to-information-act-
2005/#:~:text=The%20Right%20to%20Information%20Act%20was%20passed%20by%20Parliament%2
0on,functionaries%20more%20accountable%20and%20responsible.
99. Ananya was a student of CD Goenka School in Shastri Nagar. CD Goenka ran under a private entity i.e.,
Goenka Family. Ananya appeared for her 11th standard exam for which she prepared really well but got less
than 40% in her final exams. Ananya got frustrated after seeing her result and wanted to file an RTI before her
school to request her school authorities to show her answer scripts. Can she do so? Decide.
a) Yes, because it her right to see her answer script.
b) Yes, because it comes under the definition of Sections 2(f) and 2(j) of the RTI Act.
c) Both A and B taken together
d) None of these
100. In the facts of above question, assume that CD Goenka was administered under the aegis of the state
government and Ananya wanted to file an RTI to request her school authorities to show her answer scripts.
Can she do so?
a) Yes, because it her right to see her answer script.
b) Yes, because it comes under the definition of Sections 2(f) and 2(j) of the RTI Act.
c) Both A and B
d) None of these
101. Vijay was a civil servant and was posted in New Delhi. Vijay‘s son Ajay got married recently and it was a grand
event. Suraj, an RTI activist, was an enemy of Vijay. Suraj filed and RTI demanding Vijay to disclose the details
of gifts he received by him and his family at the wedding of his son.. Will Suraj succeed? Decide
a) No, Suraj will not succeed because he is doing everything in enmity.
b) Yes, Suraj will succeed because Vijay is a government servant.
c) No, Suraj will not succeed because the information he demanded is personal information.
d) Yes, because a public servant has to disclose each and every asset that belongs to him and his family.

27
CLAT 02 (2022 – 23)
102. Central Government appointed Directors in nine IIMs. Virat, one of several applicants, filed an RTI application
demanding information regarding the appointment of nine Directors in IIMs. The information was released
with the seniority-cum-merit list but the personal information of the selected candidates was rejected on the
grounds of exemption provisions under Section 8. Virat was not satisfied with the information and filed legal
proceedings against the respective government department. Will Virat succeed? Decide.
a) Virat will not succeed because necessary information was already given to him.
b) Virat will succeed because the information does not come under section 8.
c) Virat will not succeed because the further information will come under the provision of personal privacy.
d) None of these
103. Aman was working in an MNC. Aman suddenly died of a heart attack. Misha, his daughter, went to the Tehsil
Office several times to get her father‘s death certificate because it was needed to complete formalities like
insurance claim, funds, etc. However, every time she was denied by the officer there without giving any
reason. Misha wants to file an RTI for getting access to the document(s) rejecting her requests for supply of of
death certificate. Can she do so? Decide.
a) No, because the information related to dead person cannot be inquired under RTI Act.
b) Yes, because her request falls under the ambit of the RTI Act.
c) No, because her request does not fall under the ambit of the RTI Act.
d) No, because she has other means to inquire about the same and first she has to exhaust those means.
104. The RTI application is addressed to __________?
a) President
b) Vice-President
c) Ministry of Information and Broadcasting
d) Public Information Officer
105. RTI is a facet of which fundamental right?
a) Right to equality and transparency
b) Right to accountability
c) Right to freedom of speech and expression
d) Right against exploitation

Direction for Questions 106 to 110: Read the passage and answer the following questions.
That lockdown is not a necessary condition for victory is emphatically supported by the fact that more than a
100 countries practised lockdowns at one time or another, and less than a dozen succeeded. But all countries
that did succeed also practised ancient wisdom as embedded in the 3-T policy. This fact deserves emphasis.
The world has endured many viruses, many contagions. Check your history books — no region/country
practised a lockdown prior to 2020. Should one believe, therefore, that all other viruses were less contagious?
The US had a version of the common flu much worse than any experienced before, or since. Yet the US
authorities, under the able leadership of D A Henderson, faced the flu without lockdowns. What was the
strategy employed?
The same as that practised with all infectious diseases for centuries — attempt to isolate, and social distance
from the patient. In the 1957-58 flu, some schools were closed, others remained open; ditto with workplaces;
but emphatically no lockdown. The 3-T is a modern version of the same. The deaths per million evolution is
like cases, but there is a noticeable difference. Deaths are considerably lower in India — only 113 per million
versus 271 per million for the world and 274 for MENA. Note the very low value of deaths in sub-Saharan
Africa and in East Asia. Two broad conclusions follow, but also need a follow-up.
First, poorer countries seem to have been affected less adversely than the richer, more advanced economies.
One suggested explanation (possibility) is that poorer countries have had to live longer with diseases (and bad
sanitation) and, therefore, have developed some immunity which make them less prone to infections, ceteris
paribus. Second, East Asia has a very low incidence of COVID-19, even after excluding China. One possible
explanation is that this region has had greater experience with flu (for example, the avian flu).
One additional note about the continuing march towards herd immunity in India. In a paper presented at
IRADE in December, we made a forecast of the ―end‖ of COVID in India — we had said 12 million infections
and the end sometime in April. Our forecast for India as of today — 10.5 million infections and the end
28
CLAT 02 (2022 – 23)
sometime in May-June. Based on the available state-level data, some states in India have already achieved herd
immunity while other states are catching up. States like Andhra Pradesh, Assam, Bihar, Jammu and Kashmir,
Jharkhand and Goa have seen flattening; others like Karnataka, Maharashtra, are catching up.
One final note: Can we infer from these data that India might be approaching herd immunity, and doing so
before vaccines are widely available? The many East Asian economies that have succeeded (for example,
Vietnam, Taiwan, Cambodia, Laos, China etc.) did so by controlling infections at an early stage, that is,
infections were never large enough to warrant a relationship with herd immunity. Ditto the case with regions
in sub-Saharan Africa. Only India stands out with a flattening curve among those that had earlier experienced
an explosion of infections. The extraordinary decline of COVID-19 cases (and deaths) in India is an issue, like
lockdowns, deserving of a more detailed study.
[Source: https://indianexpress.com/article/opinion/columns/india-coronavirus-herd-immunity-7148062/ ]
106. The author argues that lockdowns are not necessary to fight the virus and gives an empirical example for the
same. Which of the statements below, if true would weaken the author‘s argument?
a) The countries which imposed lockdowns witnessed a slow growth in the virus.
b) Lockdown is required because people fail socially distance and wear masks in public.
c) The countries where lockdown was not successful did not follow not wearing masks policy.
d) Lockdown is an inhibitor not a killer of virus.
107. Which of the following is NOT true according to the passage?
a) India has witnessed lower number of deaths per million because of its large population.
b) Indians have a higher immunity and hence are able to fight off the virus well.
c) Insanitary conditions in which Indians have lived for long have made our bodies capable of withstanding
diseases.
d) All of the above.
108. One suggested explanation (possibility) is that poorer countries have had to live longer with diseases (and bad
sanitation) and, therefore, have developed some immunity which make them less prone to infections. What
assumption does the author make in the above statement?
a) Higher immunity wards off diseases faster.
b) The fact that all poorer countries have fared well in the fight against coronavirus shows that high immunity
is the common rationale.
c) If you live with in poorly sanitised conditions your immunity system is more effective.
d) None of the above.
109. The author states that US faced the common flu without lockdowns. What is the reason behind this success?
a) Lockdown is not a necessary condition to fight an infectious disease.
b) Lockdowns are futile if no one social distances.
c) Patients were isolated and people socially distanced themselves from such patients.
d) None of the above.
110. What is the main idea of the passage?
a) The passage describes the futility of lockdowns.
b) Discussing the importance of social distancing and finding the reasons behind India‘s low death per million
numbers.
c) Arguing that lockdown is not necessary and suggesting that a study be conducted into India‘s lower death
per million numbers.
d) The passage draws a connection between lack of lockdown and control of coronavirus.

Direction for Questions 111 to 115: Read the passage and answer the following questions.
It feels like life has come a full circle as we are back in the month of January and China is back with a
lockdown. The last one year – in many ways for most of us can be considered as a zero year. But it was a zero
year with significant discussions on the pandemic, its likely progression, its impact on inflation and so on and so
forth. Let us focus on inflation and the conventional arguments that were raised during the course of the year,
including those by a former Governor and Deputy Governor raising concerns of ‗persistent and high inflation‘
as a consequence of the monetary easing. Many presented an argument that the rate cuts would result in

29
CLAT 02 (2022 – 23)
inflationary impulse becoming permanent, which would then feed into inflationary expectations. This would
have, as per many, resulted in an inflationary spiral, which would require aggressive tightening in the future.
The opposing view – and one that I ascribe to - was that there would be some temporary inflation as an
outcome of the Covid-induced disruption in supply chains which would repair over time. Thus, inflation would
be driven by non-monetary factors and the real threat was of an income shock in subsequent periods which
could push down prices. Inflation has since then eased as it is back in the 2-6 per cent range and it will perhaps
moderate over the coming few months and might be even below 4 per cent for a prolonged period.
One of the key reasons why I believed that there are limited prospects of an inflationary impulse was because
nominal wages were slashed. Conventional theory suggests that inflation picks up when the economy
overheats, and demand far exceeds the available supply of goods thereby pushing prices. When wages are
down, then the chances of economy overheating are limited - thus if there is inflation in such situations, then it
must be because of supply factors such as an increase in cost of inputs. An example for this is the recent
increase in container prices as global trade is a fraction of what it used to be before. The increase in container
prices is pushing up the cost of importing raw materials in some sectors and is an example of supply
constraints pushing up prices.
In such a case, increasing interest rates could have been counterproductive. Therefore, we must applaud the
Monetary Policy Committee (MPC), and primarily Dr Ashima Goyal for repeatedly stressing on the fact that
inflation in preceding months was temporary. The reason why this discussion is important is because inflation
being well within the 2-6 per cent range has interesting implications for all of us. What this means is that MPC
can continue to keep the interest rates low in the foreseeable future without any looming threat of inflation
becoming a key concern.
In fact, MPC had explicitly stated that it will continue its accommodative stance till the economy recovers –
but with inflation moderating, there are prospects of the RBI keeping rates low well beyond our economic
recovery as inflation could well be muted. The simple argument here is that if there is no inflationary impulse,
then why would the MPC hike rates? After all, its mandate is to achieve the target of 4 per cent inflation (+/-2
per cent). Lower rates are critical as they affect sectors such as real estate and automobiles given that EMIs
are essentially the instrument through which people evaluate the cost of their ownership. The recent pickup in
residential real estate is largely an outcome of the confluence of stamp duty cuts and lower rate of interests
which have made home ownership an attractive proposition.
Thus, clearly low rates and surplus liquidity is assisting our economic recovery while also creating space for the
government to borrow to meet its deficit. One of the institutions which I criticised a lot in the past has
emerged to be a star-performer for India during the pandemic – and that is the Reserve Bank of India. Here‘s
hoping that the measured approach of evaluating the drivers of inflation before responding using monetary
policy tools becomes a key institutional mechanism to ensure such a performance for the future.
[Source: https://swarajyamag.com/economy/and-where-is-the-bogey-of-inflation ]
111. The author argues that inflation occurs as a result of demand-supply deficit. Which of the following statements
if true strengthen the author‘s argument?
a) When people have more money to spend, the value of money goes down and hence, inflation occurs.
b) Higher demand leads to higher prices.
c) X mobile‘s prices have dropped this year as its supply-chain is restored, even if the demand continues to be
the same.
d) A high demand would ultimately lead to inflation as it would put more money in the economy.
112. What is the author‘s assumption behind slashing of nominal wages?
a) Lesser amount of money in people‘s hands would translate to lower demand.
b) Lower demand would lead to prices falling and hence, there would be no inflation.
c) Lower demand stemming from lower salaries would help in controlling inflation.
d) Both A and B
113. What is the rationale behind rising demand in real estate, as per the author?
a) During inflation, demand increases and hence the demand for real estate also increased.
b) Real estate is considered a safe investment and hence, the demand from the same increased, even if the
economy plummeted.
c) Lower interest rates and low stamp duties has made real estate a good investment.
30
CLAT 02 (2022 – 23)
d) None of the above.
114. Which of these is false according to the author-
a) Inflation cannot be controlled by interest rate cuts.
b) Inflationary impulse is a permanent phenomenon.
c) Supply-chains were severely affected during COVID and this led to a demand-supply deficit, hence pushing
inflation.
d) Both A and B
115. What is the core argument given by the author?
a) Inflation can be controlled by non-monetary tools if the drivers of inflation are controlled.
b) Demand-supply deficit is the true cause of inflation.
c) COVID-19 caused temporary disruptions in the demand-supply chains.
d) All of the above

Direction for Questions 116 to 120: Read the passage and answer the following questions.
Big Tech is not stupid. They understand that their business is different from usual free enterprise. The
difference lies in the ability to scale. To double the size of your shirt business, it would take huge amounts of
time and capital investment. You can‘t just double the size of your factory like that. But a tech company? If it
does well, it could go from 10 users to 10 million users in a few months. Hence, the valuation.
This is what makes Big Tech censorship so frightening. The ability to scale. Traditional censorship, even under
an oppressive regime such as Iran, suffers from an inability to scale. How many dissenters could Iran possibly
throw into jail? But with a few tweaks to its algorithm, Big Tech could profile millions of people who have a
particular political ideology. It could shut down their accounts instantly. It could share this data with other
companies, with banks, phone operators and employers. You would lose your job, your bank account, even
your cellphone connection. Literally hundreds of millions of dissenters could be silenced in a single day. And
nobody would know of their fate because the news could be filtered out too. This kind of suppression has
never been possible in history. That‘s why you can no longer apply the usual way of thinking about regulation
and private enterprise to Big Tech. The scale of Big Tech beats anything that could have been imagined when
we laid down those principles.
It is similar to the second amendment debate they have in the US. The US Constitution gives people the right
to bear arms. But which arms? Would it allow a private citizen to own a nuclear bomb? Those who framed the
US Constitution in the 1700s could never have visualized such weapons. They were probably just thinking
about guns and rather primitive ones at that. That‘s why you can‘t apply the second amendment to today‘s
inter-continental ballistic missiles.
What happens next? At least three things are possible. One, we could really be headed for a dictatorship of Big
Tech. They would tell you what to think and when. All other thought would be rendered impossible, even
subconsciously. Second, Big Tech could sell us all out to China. Big Tech has a big problem too. To start building
their power, they need a democratic society to begin with. China would never allow Big Tech to build itself up
to a position from where they could challenge government authority. They would send the tech giants into
―enforced supervision‖ long before that. Third, social media could split along ideological lines, like mainstream
media.
In conclusion, the right should not make the same mistake with Big Tech that the left made with the
knowledge economy. The way the left frames issues has remained frozen in the times of the industrial
revolution. Asking for things like fixed workdays and fixed work hours in return for fixed compensation makes
sense if your work is shoveling coal into a locomotive engine. When the economy became full of knowledge
workers, the demands of the left stopped making sense. How would you tell a computer programmer to have
ideas only between 9 AM and 5 PM five days a week, with a one hour break for lunch? The left began to lose
relevance on economic issues because all their ideas were so outdated. At the moment, the same could be
happening to the right. The right is still stuck in time, framing issues in terms of rights of free enterprise. Those
issues don‘t make sense in a world dominated by Big Tech. It is time for the right to rethink everything it
stands for.
[Source: https://www.opindia.com/2021/01/big-tech-censorship-and-the-three-things-that-can-happen/ ]
116. Summarily this passage wants to convey which of the following?
31
CLAT 02 (2022 – 23)
a) Big Tech has the capability of becoming dictatorial.
b) The Right-wingers should understand how rights of free enterprise do not apply to Big Tech.
c) Big Tech should be controlled before it becomes too big to control.
d) Right-wingers should alter their ideology to understand that rights of free enterprise cannot be applied in a
blanket manner.
117. The author argues that the ability to scale makes the Big Tech frightening. Which of these statements if true
weakens the Authors argument?
a) People using technology in the world is still very low and hence the reach of Big Tech is limited.
b) People have an option to delete their accounts, hence, the ability to scale upwards also comes with a risk of
scaling downwards.
c) Political systems of many countries such as China heavily regulate Big Tech.
d) None of the above.
118. Which of the following statements weaken the core argument being made by the author?
a) Regulating Big Tech which is heavily based on common people may lead to muzzling of voices of people.
b) A wide public outreach and an ability to scale cannot be the basis of not treating Big Tech as another free
enterprise.
c) Big Tech always gives an option to its users to not use its services, this amounts to Big Tech not being
dictatorial.
d) All of the above.
119. The author states that Big Tech requires a democratic society to set up. What does the author imply through
this statement?
a) Democracies value the rights of free enterprise.
b) Democracies allow their people to choose which technology they want to use.
c) Both A and B
d) In a Democratic Society you can critique the government without any consequences.
120. “social media could split along ideological lines, like mainstream media.” What is the assumption underlying the
given statement?
a) All media are susceptible to be divided along ideological lines.
b) Mainstream media has ideological divides.
c) Social media is prone to be divided on ideological lines.
d) Ideological lines are bound to occur wherever mass communication happens.

Direction for Questions 121 to 125: Read the passage and answer the following questions.
For most Indians, the news of a siege on Capitol Hill in Washington DC by a belligerent and raucous mob of
Trump supporters, caused shock and disbelief, as visuals of the Capitol — engulfed in smoke, a deserted
Senate floor, members of the House of Representatives taking cover, and armed ruffians vandalizing the halls
of that chamber of democracy — began to circulate widely.
The Indian government swiftly condemned the incident, with the prime minister tweeting that he was
―distressed to see news about rioting and violence in Washington DC. AN orderly and peaceful transfer of
power must continue. The democratic process cannot be allowed to be subverted through unlawful protests‖.
But these words, though timely, and all the more welcome as a retreat from ―ab ki baar Trump Sarkar‖ — his
inappropriate pronouncement in Houston — are likely to do little to assuage the growing concerns of many
Indians who see our own country tread a similar path.
The stratification of our two societies has been accelerated by the dominant political leadership of the two
countries. Both Trump and Modi have fashioned themselves as the voice of the people, promising to restore
lost national pride, increase economic growth and take back traditional power centers that had been
dominated by the liberal elites. Trump‘s key slogans like ‗America First‘ and ‗Make America Great Again‘ had
counterparts in Modi‘s ‗sabka saath, sabka vishwas’ and the promise of ‗acche din‘ and a ‗Naya Bharat’. Where
Trump promised to get rid of the Washington elite and ‗drain the swamp‘, our own ruling dispensation labelled
their adversaries as the ‗Lootyens elite‘ and the ‗Khan Market Gang‘. With their assiduously built-up cults of
personality, both offered ‗strong leadership‗ and successfully persuaded their voters that they were more

32
CLAT 02 (2022 – 23)
authentic embodiments of their nations than the allegedly rootless secular cosmopolitans they sought to
displace.
Institutions have been enfeebled in both nations but the assault on our institutions has been far more direct,
including pressure on financial regulators like the RBI; the investigative agencies (notably the Central Bureau of
Investigation); the Election Commission, which organises, conducts, and rules on the country‘s general and
state elections; the upper echelons of the Armed Forces; institutions of accountability like the Central
Information Commission; the elected legislatures; the judiciary, headed by the Supreme Court — and even the
free press.
The result is the decay of the liberal, tolerant and democratic ethos that both countries have been proud to
share. It is our task in India to rebuild that original ethos that animated our freedom struggle and characterized
our inclusive nationalism. Let the events in Capitol Hill serve as a stark warning to us of what could come our
way if we fail to do so.
[Source: https://www.thequint.com/voices/opinion/united-states-capitol-siege-trump-supporters-white-
supremacy-modi-govt-hindutva-lessons-for-india#read-more ]
121. The author states that many Indians think that our nation is on a similar path as the US where a siege was laid
on the capitol building. What rationale does he give for the same?
a) India has a leader who, like Trump promises to be the voice of the people by restoring lost pride.
b) Public Institutions have been enfeebled and all accountability is lost.
c) Both the leaderships are against elite culture.
d) All of the above.
122. The author argues that both the leaders give similar slogans hence India is on the same path as the US. Which
of the statements if true, would weaken the author‘s argument?
a) Sloganeering is of little importance unless it is coupled with actions.
b) Promising to restore the pride of the country does not amount to sloganeering.
c) Similar slogans does not mean that the public would react in the same way.
d) All of the above.
123. With their assiduously built-up cults of personality, both offered ‗strong leadership‗ and successfully persuaded
their voters that they were more authentic embodiments of their nations than the allegedly rootless secular
cosmopolitans they sought to displace. What assumption underlies the given statement?
a) Strong leaderships have persuasive powers.
b) People tend to believe those with strong personalities.
c) Artificial creation of personality cults often leads to people believing in the leadership qualities of such
personalities.
d) Perception of a Personality cult offering strong leadership make people believe anything with ease.
124. Which of the following is NOT true according to the passage?
a) India may see an attack on its democratic institutions as the current ruling dispensation is similar to that of
Trump government.
b) India‘s institutions have enfeebled a lot more than US institutions.
c) PM‘s statement condemning the siege of Capitol Hill has repaired the damage done, when he had given the
slogan supporting Trump during elections.
d) All of the above.
125. What conclusion can you derive from the passage?
a) We need to rebuild our nation and bring back our lost pride.
b) We should be forewarned that the events at the Capitol can unfurl in India too.
c) We should revive liberal and democratic principles if we want to avoid a similar incident that of Capitol.
d) None of the above.

Direction for Questions 126 to 129: Read the passage and answer the following questions.
No tax is popular. But one attracts particular venom. Inheritance tax is routinely seen as the least fair by
Britons and Americans. This hostility spans income brackets. Indeed, surveys suggest that opposition to

33
CLAT 02 (2022 – 23)
inheritance and estate taxes (one levied on heirs and the other on legacies) is even stronger among the poor
than the rich.
Politicians know a vote-winner when they see one. The estate of a dead adult American is 95% less likely to
face tax now than in the 1960s. And Republicans want to go all the way: the House of Representatives has
passed a tax-reform plan that would completely abolish ―death taxes‖ by 2025. For a time before the second
world war, Britons were more likely to pay death duties than income tax; today less than 5% of estates catch
the taxman‘s eye. It is not just Anglo-Saxons. Revenue from these taxes in OECD countries, as a share of total
government revenue, has fallen sharply since the 1960s. Many other countries have gone down the same path.
In 2004, even the egalitarian Swedes decided that their inheritance tax should be abolished
Some people argue for a punitive inheritance tax. They start with the negative argument that dead people no
longer enjoy the general freedom to disburse their wealth as they wish—as the dead have no rights. How
could they, when they are not affected one way or the other by what happens in the world?
The positive argument for steep inheritance taxes is that they promote fairness and equality. Heirs have rarely
done anything to deserve the money that comes their way. Liberals, from John Stuart Mill to Theodore
Roosevelt, thought that needed correcting. Roosevelt, who warned that letting huge fortunes pass across
generations was ―of great and genuine detriment to the community at large‖, would doubtless be aghast at
the situation today. Annual flows of inheritance in France have tripled as a proportion of GDP since the 1950s.
Half of Europe‘s billionaires have inherited their wealth, and their number seems to be rising.
The right approach is to strike a balance between the two extremes. The precise rate will vary from country
to country. But three design principles stand out. First, target the wealthy; that means taxing inheritors rather
than estates and setting a meaningful exemption threshold. Second, keep it simple. Third, with the fiscal
headroom generated by higher inheritance tax, reduce other taxes, lightening the load for most people.
Source: https://www.economist.com/leaders/2017/11/23/a-hated-tax-but-a-fair-one
126. Which of the given options with which the author of the passage will agree to?
a) None of the given options
b) Government revenue from inheritance taxes is steeply rising by the decade.
c) Inheritance tax is beyond the purview of political debates.
d) The government of Sweden does not believe in an egalitarian society.
127. Which of the following is/are the argument(s) for implementing inheritance tax?
I. A high rate of inheritance tax is crucial for creating a fair as well as an egalitarian society.
II. The heirs live in a certain state and the state has every right to tax them in any way possible.
III. The heirs of the inheritance rarely do something outstanding to deserve the wealth from their
predecessors.
a) Only I
b) Only II
c) Both I and III
d) Both II and III
128. Which of the following statement (s) is/are TRUE as per the passage?
I. Individuals such as John Stuart Mill and Theodore Roosevelt staunchly opposed the inheritance tax.
II. Today, fewer Britons pay death duties compared to the Britons who lived before the second world war.
III. The politicians are trying to completely remove the inheritance tax or ―death tax‖ by the year 2025.
a) Only I
b) Both I and II
c) Only III
d) Both II and III
129. According to the passage, which of the following argument is likely to be similar with the viewpoint of the
author?
a) The complete abolition of the inheritance tax is necessary as the dead does not really care about what is
done with their inheritance.
b) A steep inheritance tax is required because the money might fall into the hands of someone immoral,
resulting in a detrimental society.
c) Tax only the rich inheritors as that will lead to an egalitarian society.
34
CLAT 02 (2022 – 23)
d) A middle ground between the two extremes must be found which will be equitable towards all inheritors.

Direction for Questions 130 to 135: Read the passage and answer the following questions.
The portrayal of blackface–when people darken their skin with shoe polish, greasepaint or burnt cork and
paint on enlarged lips and other exaggerated features, is steeped in centuries of racism. It peaked in popularity
during an era in the United States when demands for civil rights by recently emancipated slaves triggered racial
hostility. And today, because of blackface‘s historic use to denigrate people of African descent, its continued
use is still considered racist.
Although the exact moment when blackface originated isn‘t known, its roots date back to centuries-old
European theatrical productions, most famously, Shakespeare‘s Othello. The practice then began in the
United States in the 18th century, when European immigrants brought the genre over and performed in
seaports along the Northeast, says Daphne Brooks, a professor of African American studies and theater
studies at Yale University.‖ But the most famous sort of era to think of as being the birth of the form itself is
the Antebellum era of the early 19th century,‖ Brooks says.
Thomas Dartmouth Rice , an actor born in New York, is considered the ―Father of Minstrelsy.‖ After
reportedly traveling to the South and observing slaves, Rice developed a black stage character called ―Jim
Crow‖ in 1830.
With quick dance moves, an exaggerated African-American vernacular and buffoonish behavior, Rice founded
a new genre of racialized song and dance—blackface minstrel shows—which became central to American
entertainment in the North and South. White performers in blackface played characters that perpetuated a
range of negative stereotypes about African Americans including being lazy, ignorant, superstitious,
hypersexual, criminal or cowardly.
Some characters from ―History of Minstrels: From ‗Jump Jim Crow‘ to ‗The Jazz Singer.‘‖ Some of the most
famous ones were Rice‘s ―Jim Crow,‖ a rural dancing fool in tattered clothing; the ―Mammy,‖ an overweight
and loud mother figure; and ―Zip Coon,‖ a flamboyant-dressed man who used sophisticated words
incorrectly. Most of the minstrel show actors were working-class Irishmen from the Northeast, who
performed in blackface to distance themselves from their own lower social, political and economic status in
the United States, says Leonard. ―They did it to authenticate their whiteness,‖ he says
Blackface minstrel shows soared in popularity, in particular, during the period after the Civil War and into the
start of the 20th century. The widespread demeaning portrayals of African Americans paralleled a period
when southern state legislatures were passing ―black codes‖ to restrict the behavior of former slaves and
other African Americans.
As society modernized, so did the ways in which blackface was portrayed. Not only was blackface in theaters,
but it moved to the film industry. In the blockbuster movie The Birth of a Nation, blackface characters were
seen as unscrupulous and rapists. African Americans protested the film‘s portrayals and its distorted take on
the post-Civil War era, yet it continued to be popular among white audiences.
African-Americans also performed in blackface given it was the only way to be in the entertainment industry.
But their performances countered some of the primitive representations that were popularized. But blackface
minstrelsy remained a genre heavily dominated by white actors.
Source: https://www.history.com/news/blackface-history-racism-origins
130. What is the purpose of the passage?
a) To show how blackface framed a version of Africans in our minds.
b) To show how the history of blackface is rooted in racism.
c) To show the impacts that blackface has today.
d) To show how blackface gave an opportunity to blacks to enter the entertainment industry.
131. Which of the following would exemplify that ― they did it to authenticate their whiteness‖?
a) We can become the other and mock the other and assert our superiority by dehumanizing the other.
b) We can become the other and appreciate the other and assert our superiority by humanizing the other.
c) We can become the other and mock the other and assert our superiority by humanizing the other.
d) We can become the other and appreciate the other and assert our superiority by dehumanizing the other.
132. Why did the author mention various characters of minstrel?
a) To show the diversity among the characters.
35
CLAT 02 (2022 – 23)
b) To show the importance of each and every character.
c) To show the growth of the characters.
d) To show several characters became the archetype.
133. Why has the author mentioned some recent examples in the last paragraph?
a) To highlight that it is still an ongoing issue.
b) To highlight that it will never end.
c) To highlight the plight of blacks.
d) To highlight the necessity of rules.
134. What is the structure of the passage?
a) Cause is being discussed and then concluded with a solution.
b) Origin is discussed and then concluded with a present scenario.
c) Impact is discussed and then concluded with a present scenario.
d) Origin is discussed and the concluded with another discussion.
135. As per the passage, we can conclude that Minstrelsy was:
a) An European form of entertainment.
b) An American form of entertainment.
c) A form of entertainment developed in the early 18th century.
d) a form of entertainment which only consisted of African-Americans as the participants.

Directions for Questions 136 to 140: Study the given information carefully to answer the questions.
Three friends Seeta, Reeta and Geeta spends and their monthly salary on travelling in the
given order and each of them save half of the remaining amount. The monthly salary of Seeta and Geeta is
same and the monthly saving of Seeta is Rs more than that of Geeta. The total expenditures of Seeta and
Reeta together on travelling is Rs more than that of Geeta.
136. What is the monthly expenditure of Seeta and Geeta together on travelling?
a) Rs.
b) Rs.
c) Rs.
d) Rs. 4480
137. The monthly salary of Reeta is how much more than/less than that of seta?
a) Rs. less
b) Rs. 3000 more
c) Rs. 4000 more
d) Rs. 3000 less
138. What is the sum of the saving of all the three friends together?
a) Rs.
b) Rs. 22480
c) Rs. 24000
d) Rs. 20800
139. The total monthly saving of three friends together ¡s what percentage of their total monthly salary?
a) 42%
b) 44%
c) 41%
d) 43%
140. By how much should Seeta‘s monthly salary be increased so the monthly expenditures of Seeta on travelling
will become equal to that of Geeta?
a) Rs. 4000
b) Rs. 6000
c) Rs. 8400
d) Rs. 3600

36
CLAT 02 (2022 – 23)
Directions for Questions 141 to 145: Study the given information carefully to answer the questions.
Three friends, Chand, Chandni, and Chanchal went to a shopping centre. Each of them had Rs.2500. In the
shopping centre, the session sale discount was 20% on the marked price. Chandni and Chanchal were regular
customers so they got 25% each an additional discount on the discounted price but Chand being a new
customer didn‘t get any additional discount. Only Chanchal had a membership card of the shopping centre
which gave an additional discount of 30% on the discounted price. They all like Juicers of xyz brand and they
purchased one piece each of that brand. The marked price of each piece was same. In last, when they
calculated then they found that Chandni had paid Rs.540 more than that of Chanchal.
141. If all of them combine the money paid for Juicer then, the total money paid by them for three pieces of the
juicers was what percentage of the total marked price of the three juicers?
a) 62%
b)
c)
d)
142. The amount paid by Chand for the juicer was how much more than that by Chanchal?
a) 45%
b) 50%
c) 55.33%
d)
143. What is the ratio of the amount paid by Chand to that by Chandni?
a) 9:7
b) 3:2
c) 6:5
d) 4:3
144. How much money was left with Chanchal after purchasing thejuicer?
a) Rs. 900
b) Rs. 500
c) Rs. 1240
d) Rs. 750
145. What was the average of the amount paid by Chand and the marked price of all the juicer?
a) Rs. 2700
b) Rs. 2400
c) Rs. 2000
d) Rs. 2150

Directions for Questions 146 to 150: Study the given information carefully to answer the questions
Every year, a survey of people is conducted by the organization (WHO). WHO found that in the year
the percentage of people affected by malaria were
. WHO also found that every year out of the affected people 60% were
students, were house-wives and were drivers. The number of house-wives, students and drivers
were in the ratio every year.
146. In the year 2003 find the number of house-wives affected by malaria?
a)
b)
c)
d)
147. In the year 2005, find the number of drivers who were not affected by malaria?
a)
b)
c)
d)

37
CLAT 02 (2022 – 23)
148. What is difference in the number of students affected and not affected by malaria in the year 2002?
a) 20
b) 30
c) 100
d) 210
149. Find the ratio of the number of house-wives affected by malaria in the year 2001 to that affected by malaria in
the year 2004.
a) 5:3
b) 9:4
c) 1:1
d) 2:1
150. Which year had the minimum number of students not affected by malaria?
a) 2001
b) 2002
c) 2004
d) 2005

38

You might also like